BarBri MBE Preview Diagnostic

Réussis tes devoirs et examens dès maintenant avec Quizwiz!

A shopper was buying groceries and attempted to pay with a personal check. The grocery clerk thought that she recognized the shopper's name from a store memo listing known habitual bad check writers in the area. The clerk immediately called for the store's security guard over the store's loudspeaker. The guard quickly approached the shopper and requested that she step into his office. The shopper protested her innocence but accompanied the guard. He told her not to leave or he would call the police. She was then kept waiting in the guard's office for an hour until the guard finally returned with a copy of the memo showing that the shopper's name was not on the list. The shopper was terribly embarrassed by the entire incident because a number of her neighbors had seen the guard take her away. If the shopper sues the grocery store for damages for her humiliation, will she prevail? (A) Yes, because she was falsely imprisoned. (B) Yes, because the grocery store was negligent in identifying her. (C) No, because humiliation is not actionable. (D) No, unless the grocery store's conduct is judged to be extreme and outrageous.

(A) Because the shopper was falsely imprisoned, she can recover damages for the humiliation she suffered. False imprisonment requires an act or omission that confines or restrains the plaintiff to a bounded area, intent, and causation. The shopkeeper's privilege does not apply because the length of detention must be for a reasonable period of time only, for the purpose of making an investigation, and here the detention of an hour to check her name on a list was clearly unreasonable. Hence, the store is vicariously liable for the actions of its security guard. (B) is not quite an accurate statement. The fact that the shopper was falsely imprisoned may be attributable to the fact that the grocery store was negligent in identifying her, but her cause of action arises out of the fact that she was falsely imprisoned, regardless of how it came about. (C) is incorrect; once the tort of false imprisonment is established, humiliation is a type of actual damages that can be recovered. (D) is an incorrect statement because, while the grocery store's outrageous conduct might be important with regard to the shopper's right to recover for intentional infliction of emotional distress, she can recover humiliation damages because of the false imprisonment even without that showing.

A pregnant woman visited her family doctor to seek relief from arthritis in her hips. The doctor prescribed a steroid-type drug. The pregnant woman took the drug as prescribed and obtained relief from her pain. The pharmaceutical company which manufactured the drug enclosed information concerning the drug in the packages it shipped to drugstores and with samples provided to physicians. This literature warned that the drug should not be prescribed for pregnant women because it caused enlargement of the ears of male children. The enlargement occurred when the children reach puberty. Studies confirming this result were published in various medical journals. The pregnant woman gave birth to a son soon thereafter. Although the son had a normal early childhood, his ears grew disproportionately large when he reached age 16, and he has suffered severe emotional distress because of his appearance. Now 17, the son has brought suit against the doctor for malpractice. In that lawsuit, which party is likely to prevail? (A) The son will prevail if he can prove that most doctors would not have prescribed the drug for his mother. (B) The son will prevail unless the doctor establishes that he had not read the journals reporting on the side effects of the drug. (C) The doctor will prevail because he qualifies as a "learned intermediary" with regard to the warning about the drug from the pharmaceutical company. (D) The doctor will prevail because no duty is owed to an unborn child.

(A) If the son can prove that "most doctors" would not have used this drug to treat a pregnant woman, then he has probably shown that the doctor has failed to comply with the required standard of care. The mother's doctor owes a duty to both mother and child not to act in a negligent manner. If a family practitioner practicing according to the standard of care required in this jurisdiction would not have treated an obviously pregnant woman with the drug, then the doctor's treatment of the pregnant woman was negligent. (B) is incorrect. The doctor can only prevail if he can prove that treating the pregnant woman with the drug was in accord with prevailing medical procedure as practiced by a physician of ordinary care and skill. The fact that the doctor was not personally aware that the treatment he prescribed would cause injury will not allow him to avoid liability if his conduct was not in accord with the standard of care required of doctors in this jurisdiction. If he needed to read certain journals to acquire the knowledge and skill of a competent physician, then failure to do so is failure to exercise reasonable care. (C) is incorrect because the "learned intermediary" rule would protect the pharmaceutical company from being sued for failure to warn the mother about the risks (because it provided a warning to the prescribing physician); it does not affect the doctor's liability. (D) is incorrect because a fetus who is born alive has a cause of action against those who negligently cause prenatal injuries.

A privately owned bus company operated over fixed city routes under a franchise granted to it by the city council. To enhance revenues, the bus company rented advertising space on its vehicles. Although the city received a certain fixed percentage of the fare revenues from the company, it received nothing from the advertising fees. The management of the bus company had learned through bitter experience that any advertising that could be characterized as "controversial" led to complaints from riders and sometimes to losses in revenue. To that end, management employs a firm "no controversial advertising" policy. Thus, they refused to take ads for X-rated movies, abortion clinics, political candidates, and anything else that might be deemed "controversial." A candidate for city council was waging a populist campaign for the job and among his targets were the local utilities and the bus company. The candidate demanded that he be allowed to buy advertising on the buses, but the company's advertising manager refused, citing the company ban on all political ads. The candidate filed suit in federal district court, asking that the court require the company to accept his ads. Which of the following is the company's best argument against the candidate's constitutional claim? (A) The company is a privately owned company, and thus there is insufficient "state action" to invoke the United States Constitution. (B) Commercial speech receives less protection under the First Amendment than most other forms of speech. (C) Bus riders have First and Fourteenth Amendment rights to see the type of advertising they desire, and past experience has shown that there is substantial opposition by riders to controversial ads. (D) The candidate has many other sources of advertising available and he does not need ads on the company's vehicles to get his message to the public.

(A) The Constitution is inapplicable because the bus company is a private company. The Supreme Court has ruled that the grant of a franchise is not sufficient to create state action. [Jackson v. Metropolitan Edison (1974)] Thus, there is no basis for a First Amendment claim against the bus company. (B) is wrong; the Constitution is inapplicable because of the lack of state action. Also, (B) is wrong based on the facts of the question. The speech restricted was political, not commercial. Thus, even though commercial speech receives less protection than political speech, this speech cannot properly be characterized as speech that seeks to induce business transactions. (C) is wrong because the audience's opposition to a message does not justify otherwise impermissible censorship. Also, (C) wrongly assumes that the Constitution applies. (D) is wrong because the availability of alternative avenues of expression does not justify otherwise impermissible censorship. Moreover, it also wrongly assumes that the Constitution applies.

A textile company and the local textile union were involved in a bitter labor dispute that had required the governor to use the National Guard. The Guard took up positions surrounding the company's main plant and issued orders that no more than 20 picketers could picket at any one time and that all employees who were on strike were forbidden to cross the perimeter established by the Guard. In spite of this order, union marchers approached the plant and refused to obey the commander's order to disperse. As they crossed the perimeter, the Guard was ordered to fire on the marchers. As a result, seven strikers were injured. The seven strikers brought a claim for damages against the state, which has abolished governmental immunity. Evidence indicates that the use of weapons was not necessary to protect anyone, but that the Guard used the weapons because it had previously experienced trouble in controlling civil disorders. A claim based on which of the following is the best basis for recovery by the strikers? (A) Battery. (B) Abnormally dangerous activity. (C) Negligence. (D) Strict liability in tort.

(A) The best basis for recovery is a battery action. The facts in this case indicate that the actions of the Guard were intentional. The facts also state that it was not necessary to use weapons. If it was not necessary to use weapons, it would follow that the Guard had no privilege to use deadly force, and the state would be liable to the strikers for battery. (C) is wrong because this is clearly an intentional tort. Both (D), strict liability, and (B), abnormally dangerous activities, are torts based on unintentional injuries, so even if the plaintiffs could apply these torts to this action, it would not be the best basis for recovery since, as stated above, this is an intentional tort.

Several years ago a number of employees of the state transportation department formed a workers' organization, a voluntary association incorporated under the laws of the state. The organization has grown in size over the years and has become very powerful politically. Nearly 85% of all state highway maintenance workers are members of the organization, which is supported entirely by the dues of its members. Membership is not required as a condition of employment or advancement in the state transportation department. After a vote of its entire membership on the question, a member was expelled from the organization solely because she is an active, knowing member of a group that seeks to propagate the concept of white racial supremacy. The expelled member brings suit in an appropriate court seeking reinstatement of her membership. The suit alleges that her expulsion violates rights guaranteed to her by the First and Fourteenth Amendments. Which of the following is the strongest constitutional defense that the organization may assert to this suit? (A) The First and Fourteenth Amendments do not apply to the actions of the organization. (B) Active, knowing membership in groups with overtly racist objectives is not protected by the First and Fourteenth Amendments. (C) The organization could reasonably believe that it has a compelling interest in eliminating from its membership active, knowing members of groups with overtly racist objectives. (D) The expulsion of the member took place only after a vote of the organization's entire membership.

(A) The best defense is that First Amendment rights do not apply to the organization. First Amendment rights are made enforceable against a state through the Fourteenth Amendment. However, the amendments do not prevent private individuals or organizations from encroaching upon individual civil liberties. Because there is nothing in the facts to indicate significant state involvement (state action) with the organization (granting corporate status does not suffice), the amendments do not apply to their actions in expelling one of their members. (B) is incorrect because such membership may be protected by the First and Fourteenth Amendments. (C) and (D) would have no bearing on the alleged violation of the member's rights.

A homeowner raked up numerous dead, dry leaves that had fallen on his yard and set fire to the pile, even though the wind was blowing at 15-20 miles per hour that day, and a county ordinance made it unlawful to burn leaves on any day when the wind speed exceeded 10 miles per hour. After the homeowner went into his house, the wind whipped the burning leaves into the air and deposited some of them, still burning, on the wood-shingled roof of a neighboring house, setting the house on fire. A driver was passing by and saw the burning roof of the neighbor's house. He pulled over, got out of his car and ran across the street toward the neighbor's yard so that he could warn the occupants of the house. The driver was struck by a paramedic's vehicle just arriving in response to a telephone report of the fire. The driver brought an action against the homeowner to recover for the injuries he suffered from being struck by the emergency vehicle. How should the court rule in this action? (A) For the driver, because the homeowner's negligence was a proximate cause of the driver's injuries. (B) For the driver, because the homeowner's conduct constituted negligence per se. (C) For the homeowner, because the emergency vehicle was the actual cause of the driver's injuries. (D) For the homeowner, because one who responds to a fire assumes the risk of foreseeable dangers.

(A) The court should rule for the driver. This question is best approached by the process of elimination, and a preliminary examination of (A) suggests that it is a correct answer. The homeowner's MPQ 100 preview diagnostic exam and divider P.indd 93 12/22/2015 4:13:33 PM 94. MBE PREVIEW DIAGNOSTIC EXAM negligence in burning leaves during a period of relatively high winds resulted in a fire, and it is foreseeable that third persons will attempt to rescue the victims of the tortfeasor's negligent acts. To the extent that the emergency vehicle was an intervening force, it was a dependent one (also responding to the fire) and was not abnormal or unforeseeable. Thus, the homeowner's negligence was a proximate cause of the driver's injuries. (B) is incorrect. The issue upon which the homeowner's liability to the driver turns is proximate cause; it is immaterial to that analysis whether the homeowner's negligence is established by a showing that he failed to exercise due care (indicated by the facts) or that his conduct violated a statute and constituted negligence per se (also established by the facts), which still leaves causation and damages to be established. Even if no statute were applicable to the circumstances, the homeowner would still be liable to the driver, thus (B) is not as good an answer as (A). (C) is incorrect. The homeowner's negligence was a cause in fact of the driver's injuries, because it was the homeowner's negligence which both prompted the driver to attempt a rescue of the occupants of the burning residence, and brought the emergency vehicle onto the scene. It is possible for the separate actions of two independently operating actors to combine to injure the plaintiff, and whether either of them is liable for those injuries depends on the elements of the negligence analysis, including proximate cause. Here, the homeowner breached the duty of due care, and that breach was an actual and a proximate cause of the injury to the driver. (C) is therefore inaccurate. (D) is incorrect because there is no evidence that the driver assumed the risk of being struck by the emergency vehicle. Note that the "firefighter's rule" may bar a firefighter on assumption of risk or public policy grounds from recovering for injuries while responding to a negligently caused fire; however, the rule does not apply to the driver here.

A motorcyclist had just bought a new motorcycle. One week later, while still learning how to ride the motorcycle, he saw a classmate from school whom he did not like walking along the sidewalk. He tried to scare him by swerving onto the sidewalk at a driveway, planning to swerve back onto the street at the next driveway just in front of the classmate. As the motorcyclist attempted to swerve back onto the street, the motorcycle's front tire blew because of a latent defect, causing him to lose control of the steering. He attempted to apply the brakes, but due to his inexperience, hit the accelerator by mistake. The motorcycle struck and seriously injured the classmate. The classmate sues the motorcyclist for battery. Who is likely to prevail? (A) The classmate, because the motorcyclist intended to frighten the classmate. (B) The classmate, unless the motorcyclist's negligence in hitting the accelerator was the proximate cause of the accident. (C) The motorcyclist, because he did not intend to inflict bodily harm on the classmate. (D) The motorcyclist, because the injury was proximately caused by the defective front tire.

(A) The classmate is likely to prevail. The motorcyclist intended to assault the classmate, because he drove at the classmate with the intent to scare him. Under the doctrine of transferred intent, the intent to assault is sufficient to establish a battery if a touching results. A battery requires: (i) a harmful or offensive touching to the plaintiff's person; (ii) intent; and (iii) causation. The causation element is satisfied here because the motorcyclist set in motion the force which brought about the assault and subsequent battery. (C) is incorrect because the motorcyclist's intent to commit an assault is sufficient. (B) is incorrect because the motorcyclist's potential negligence in hitting the accelerator would not be sufficient to cut off liability for his battery. (D) is incorrect because the fact that the tire may have been defective would not qualify as an intervening force sufficient to cut off the motorcyclist's liability. The classmate may have a separate claim against the motorcycle manufacturer on a products liability theory, but the motorcyclist remains liable for the battery.

A nursing home, as a matter of policy, admitted patients known for violent behavior. Each night before the patients went to bed, all doors were locked so that intruders could not enter the premises without forcing an entry. One morning an elderly and docile patient was found beaten in her bed. The doors had been locked and there were no signs of forced entry into the building during the previous night. The patient sued the nursing home for her injuries and established the above facts. At the close of the patient's case, the nursing home moved for a directed verdict. How should the court rule? (A) Deny the motion, because the jury could find that an occurrence such as this would not ordinarily happen unless the nursing home was negligent. (B) Deny the motion, because the nursing home is required to exercise a very high degree of care towards the patients in its care. (C) Grant the motion, because the patient did not establish that the beating was caused by an employee of the nursing home. (D) Grant the motion, because the beating was caused by the criminal act of a third party.

(A) The court should deny the motion because the nursing home could be found liable under the doctrine of res ipsa loquitur. This doctrine would apply because the injury ordinarily would not have occurred in the absence of negligence on the part of the nursing home, such as by accepting violent patients without providing adequate security for other patients. The nursing home has assumed the duty of ordinary care for the safety and security of its patients. Because the doors were locked, the injury occurred as a result of either a nursing home employee or a patient, and the nursing home would be liable in either case. Admitting patients known for their violent behavior created a foreseeable risk of injury to all the patients, and the exercise of ordinary care would require that the nursing home provide adequate security to protect the patients. The patient's beating establishes enough evidence of the breach of duty by the nursing home here to withstand the motion and send the case to the jury. (B) is incorrect. There is no rule of law that imposes a high degree of care in this situation—just ordinary care under the circumstances. Thus, the nursing home is only liable because it has breached its duty of ordinary care for the safety and security of the patients. (C) is incorrect. The patient need not prove that the beating was caused by an employee in order to impose liability on the nursing home, because violence by another patient was foreseeable. The nursing home has assumed the duty of ordinary care for the safety and security of its patients. (D) is incorrect. An intervening criminal act of a third party will not cut off liability if it was foreseeable and the defendant owed a duty of care to the plaintiff. The nursing home has assumed the duty of ordinary care for the safety and security of its patients, and here criminal acts were foreseeable because the nursing home accepted violent patients.

While driving in State A, the defendant, a State B resident, was in an automobile accident with the plaintiff, a resident of State A. The plaintiff filed a negligence action against the defendant in a State A state court consistent with State A's long arm statute, and properly served the State B defendant pursuant to the State A long arm statute. The State B defendant immediately filed a motion to dismiss the action on the grounds that the State A court does not have personal jurisdiction. How should the court rule on the motion to dismiss? (A) The court should deny the motion, because the defendant has purposeful contacts with State A that are directly related to the claim being asserted. (B) The court should deny the motion because the plaintiff is a State A resident. (C) The court should grant the motion unless the defendant is engaged in substantial and continuous activities such that it is essentially at home in State A. (D) The court should grant the motion because State A courts lack constitutional authority to assert jurisdiction over defendants outside of State A unless such defendants consent to the State A courts' jurisdiction.

(A) The court should deny the motion. The question provides that service was made pursuant to the State A long arm statute. The assertion of personal jurisdiction is statutorily authorized, in that it is consistent with State A's long arm statute, and the exercise of personal jurisdiction is constitutional, given that the claim arises from the defendant's purposeful activities (driving in and using roads) in State A. Thus, a court in State A may properly exercise personal jurisdiction over the defendant from State B, even if the defendant does not consent to personal jurisdiction and is not domiciled there. (B) is incorrect because personal jurisdiction cannot be exercised against a defendant based on the plaintiff's domicile. (C) is incorrect, as the answer choice describes the exercise of general jurisdiction (i.e., personal jurisdiction over the defendant for all causes of action). The facts do not indicate that the defendant has such contacts with State A. (D) is incorrect because consent is not necessary to assert personal jurisdiction over a defendant, as explained above. Consent is one basis for exercising personal jurisdiction over a defendant, but not the only basis.

A seller entered into a written land sale contract with a buyer on May 20, whereby the seller agreed to sell a home to the buyer for $60,000. The closing date was set at August 1. The buyer put up $6,000 as earnest money, as provided by the contract. The contract stated that if the buyer failed to perform by tendering the balance due on the house on August 1, the $6,000 could be treated as liquidated damages "at the option of the seller." On July 21, the week before the closing, the house burned to the ground because of a freak lightning strike during a thunderstorm. When August 1 arrived, the buyer refused to tender $54,000 to the seller. The buyer asked the seller for the refund of his earnest money because the house had been destroyed. The seller refused and filed suit, asking for specific performance. The buyer countersued, demanding refund of the $6,000 earnest money. How should the court rule on the suits? (A) The court will order specific performance by the buyer. (B) The court will order the seller to return the $6,000 to the buyer because of frustration of purpose of the contract. (C) The court will award the seller $6,000 as liquidated damages. (D) The court will award neither party the relief sought because of mutual mistake.

(A) The court should order specific performance by the buyer. Because land is considered unique, specific performance is always appropriate for the enforcement of a valid land sale contract. This option is available to either the buyer or the seller. The contract gave the seller the option of using the liquidated damages provision if the seller wished, but the seller has sued for specific performance. Because that is appropriate, it will be granted. It is not of significance that the house burned to the ground. When a contract for the sale of land is signed, equitable conversion takes place and it is, for all practical purposes, the buyer's land and the buyer's risk. Here, the risk of loss shifted to the buyer upon the signing of the contract. (B) is incorrect because frustration of purpose is not applicable. The doctrine of equitable conversion shifted the risk of loss to the buyer when the contract of sale was signed. The fact that an improvement on the property (the house) is no longer present is not relevant to the grant of specific performance. (C) is incorrect. Based upon these facts, the seller could request as a remedy either specific performance or liquidated damages of $6,000. Because the seller elected the specific performance remedy, the court will not award liquidated damages. (D) is also incorrect because there are no facts which would indicate mutual mistake. The burning down of the house would not qualify.

A suspect was captured in a high-speed police chase five minutes after a bank robbery in which the robbers had handed the teller a handwritten note demanding money. The suspect was taken to the police station where, over his protests, he was required to write out the words of the note. He was then charged with the robbery. At a pretrial suppression hearing, the suspect challenged the prosecution's plan to offer in evidence the writing that he had been required to make by the police so that the jury could compare it with the robbers' note. How should the court rule on the admissibility of the writing? (A) Admissible. (B) Inadmissible, because the suspect was not advised that his handwriting sample could be admitted into evidence against him. (C) Inadmissible, because the suspect was not advised of his right to refuse to give a handwriting sample. (D) Inadmissible, because the suspect had not been informed he had a right to have counsel present.

(A) The court should rule the writing to be admissible. The handwriting sample is relevant to the issue of the identity of the bank robber and is admissible because it was properly obtained and violated no rule of privilege. A handwriting sample is not testimonial in nature and, therefore, does not require Fifth or Sixth Amendment protections. (B) is incorrect. The handwriting sample is evidence of physical characteristics and not testimonial in nature and, therefore, not subject to Fifth Amendment protections. Therefore, there is no requirement that Miranda warnings be given, advising the defendant that the sample could be used against him. (C) is incorrect. Because the privilege against self-incrimination does not apply, there is no right to refuse to give the handwriting sample, provided that the content of the writing is not used against the defendant. (D) is incorrect. The presence of counsel is not required at a scientific identification made by the police for the purposes of investigation, such as taking a handwriting sample. [Gilbert v. California (1967)]

A developer owned an office building subject to a first mortgage with a creditor in the amount of $1 million. Subsequently, the developer borrowed $100,000 from a bank secured by a second mortgage on the building to help pay the first mortgage and other expenses of the building. The developer's financial condition worsened, and he was unable to make the required payments on the first mortgage to the creditor. The developer approached the creditor and offered to give her a deed to the building in satisfaction of all of his obligations to the creditor. The developer delivered to the creditor a quitclaim deed to the building, which recited as consideration the release of the developer from all liability on the mortgage to the creditor. The deed was duly recorded. Shortly thereafter, the office market greatly improved, and the building was worth $1.5 million. The developer then brought an action against the creditor, claiming that the deed was an equitable mortgage, and the bank served notice on the creditor that it was preparing to foreclose its mortgage on the building. Against which parties will the creditor prevail? (A) The developer only. (B) The bank only. (C) Both the developer and the bank. (D) Neither the developer nor the bank.

(A) The creditor will be successful against the developer only. The developer initiated an agreement with the creditor to convey the property to the creditor in satisfaction of the developer's obligation to the creditor. The deed was meant to convey absolute title to the creditor, and was not intended to be another form of the mortgage that already existed. Therefore, the developer has no further interest in the property. (B) is incorrect. The creditor will not prevail over the bank. The conveyance from the developer to the creditor discharged the first mortgage on the property because the conveyance was in satisfaction of the mortgage obligation. The first mortgage, however, was never foreclosed and therefore did not wipe out the second mortgage. When the first mortgage was discharged by the conveyance, the second mortgage became a first mortgage and is still a valid encumbrance on the property. (C) and (D) are incorrect because the creditor will prevail against the developer but not against the bank.

A large wholesaler of European clothing sent out a circular to all of its customers advertising fine quality coats for sale at a price of $300 each when purchased in lots of 100 coats. The circular also stated that payment for each lot was to be made in three $10,000 installments, the first payment due within 10 days of delivery and the subsequent installments within 40 and 70 days of delivery, respectively. The day after receiving the circular, a retailer sent the wholesaler a standard order form with price, quantity, and delivery date terms filled in by the retailer, clearly indicating that the retailer was ordering one lot of 100 coats. The wholesaler shipped the coats to the retailer, and they arrived on September 10. The retailer accepted delivery of the coats and, on September 11, mailed to the wholesaler a check for $29,400. The retailer enclosed a letter with the check stating, in relevant part, "By cashing the enclosed check in the amount of $29,400, we assume that you accept our standard policy of taking a 2% discount for early payment." The wholesaler received the check on September 13, read the letter, and cashed the check the next day. What is the legal effect of the wholesaler's cashing of the retailer's check? (A) The entire debt is discharged, because there has been an accord and satisfaction. (B) The entire debt is discharged on the basis of account stated. (C) The retailer remains liable for the unpaid portion of the debt, because the discount provision was never accepted as part of the contract. (D) The retailer remains liable for the unpaid portion of the debt because the check did not state "payment in full" or words to that effect.

(A) The entire debt is discharged. The initial contract was for $30,000. By cashing the check in light of the letter, the wholesaler has accepted the new payment terms. An accord and satisfaction occurs when there is a contract dispute or some slight alteration of the debtor's consideration, and here the debtor (the retailer) paid the entire contract price immediately on receipt of the coats. The acceptance of the new terms constitutes an accord; the cashing of the check represents the satisfaction. (B) is incorrect because discharge of a debt on the basis of account stated occurs when one payment is made to pay for a number of earlier invoices; that has not occurred here. (C) is incorrect because the accord is a new contract. The letter is the offer inviting acceptance by cashing the check. By cashing the check, the wholesaler accepted the discount provision. (D) is incorrect. Although an accord and satisfaction may occur through the use of such wording on a check, those words are not required when the offer of accord is communicated in another way.

A chemical plant and a steel mill located on a river both negligently discharged toxic waste into the river during the same time period. A farmer downstream who used water from the river for irrigation suffered substantial crop losses as a result of using the poisoned water. He brought suit against the chemical plant. At trial, the evidence established that the discharge from either plant alone was sufficient to have caused the farmer's crop losses. How much of the farmer's damages should he recover from the chemical plant? (A) All of his damages, because the chemical plant's negligence was a substantial factor in causing the farmer's damage. (B) Half of his damages, because he did not present evidence to allow the court to reasonably apportion damages between the two tortfeasors. (C) None of his damages, because he needed to join the other tortfeasor in the litigation. (D) None of his damages, because the harm would have occurred even in the absence of the chemical plant's negligence.

(A) The farmer should recover all of his damages against the chemical plant. To establish the element of actual cause in a negligence action, the "but for" test is ordinarily used. However, when several causes commingle and bring about an injury, and any one alone would have been sufficient to cause the injury, the "but for" test is inadequate to determine actual cause. In these cases, it is sufficient if the defendant's conduct was a "substantial factor" in causing the injury. Here, the farmer cannot establish that, but for the chemical plant's negligence, he would not have suffered damage, because either discharge was sufficient to cause his injury. However, he can establish actual cause because the chemical plant's negligence was a substantial factor in causing his injury; hence, he can recover against the chemical plant. (B) is incorrect because the farmer does not MPQ 100 preview diagnostic exam and divider P.indd 84 12/22/2015 4:13:33 PM MBE PREVIEW DIAGNOSTIC EXAM 85. need to present evidence to apportion damages. Under joint and several liability, when two or more tortious acts combine to proximately cause an indivisible injury to a plaintiff, each tortfeasor is liable to the plaintiff for the entire damage incurred. Thus, he can collect all of his damages from the chemical plant; it will be up to the chemical plant to seek apportionment of damages in a contribution action against the steel mill. (C) is similarly incorrect. The farmer does not need to join the steel mill in the litigation to recover. (D) is incorrect because that choice applies the "but for" test. As discussed above, the appropriate test in this case is the "substantial factor" test.

A writer registered under federal copyright law his copyright in certain song lyrics he wrote. The writer later entered into a contract with an advertiser in which the writer granted the advertiser a license to use the lyrics in radio advertisements. When the writer heard the advertisement using the lyrics, the writer was incensed at how the lyrics had been used. Believing that the advertiser had lied to him about how the lyrics would be used, the writer filed an action in federal district court claiming that the advertiser had made false representations that fraudulently induced the writer into entering the contract to license the lyrics. The writer is a citizen of State A. The advertiser is a partnership comprised of partners who are citizens of State A, State B, and State C. The partnership's headquarters and most of its operations are in State B. Does the federal court have subject matter jurisdiction over the action? (A) No, because the action does not arise under federal law and the parties are citizens of the same state. (B) Yes, because the plaintiff and defendant are citizens of different states. (C) Yes, because the action arises under federal law. (D) Yes, because the transaction involves interstate commerce.

(A) The federal court does not have jurisdiction because neither diversity of citizenship jurisdiction nor federal question jurisdiction exists. Diversity of citizenship jurisdiction is available when (i) there is complete diversity of citizenship, meaning that each plaintiff is a citizen of a different state from every defendant; and (ii) the amount in controversy exceeds $75,000. A natural person's citizenship is the state that is the person's domicile. A partnership is a citizen of each state of which its partners, both limited and general, are citizens. Here, the writer is a citizen of State A, and the advertiser's partners are citizens of State A, State B, and State C. Given the State A-State A connection, complete diversity does not exist. Hence, (B) is an incorrect answer choice as to diversity jurisdiction. Federal question jurisdiction is available when the plaintiff, in his wellpleaded complaint, alleges a claim that arises under federal law. Anticipation of a federal defense or the fact that federal law is implicated by the plaintiff's claim do not give rise to federal question jurisdiction; the plaintiff's claim must arise under federal law. Here, although federal copyright law is peripherally involved, the writer's cause of action is actually based on state contract law. As a result, no federal question has been presented by the writer's complaint, making (C) incorrect. Note too that federal question jurisdiction does not have a complete diversity requirement, making (B) incorrect as to federal question jurisdiction. (D) is incorrect because federal question jurisdiction does not arise merely because interstate commerce is affected.

A defendant was charged with murdering his cousin, whose body was found near the defendant's office. The prosecution alleged that the cousin, who lived in another state, confronted the defendant to tell him about an affair that the cousin was having with the defendant's wife. The defendant denied having even seen the cousin for several weeks before he was murdered. The prosecution wishes to introduce a properly authenticated letter from the cousin to the defendant's wife, dated the day before the cousin's death, that stated: "I am going to go to your husband's office and tell him face-to-face that he must let us be together." Defense counsel objects. How should the judge rule on the objection? (A) Overruled, because the statement in the letter falls under the present state of mind exception to the hearsay rule. (B) Overruled, because the statement in the letter is being offered to show the defendant's motive for murder rather than for the truth of the matter asserted. (C) Granted, because the statement in the letter is hearsay and does not fall within any exception. (D) Granted, because the cousin's state of mind is not in issue.

(A) The judge should overrule the objection because the statement falls within the exception for statements of present state of mind. Under that exception, declarations of existing state of mind are admissible as circumstantial evidence to show subsequent acts of the declarant in conformity with that intent. Here, an issue in this case is whether the cousin confronted the defendant at his office, MPQ 100 preview diagnostic exam and divider P.indd 70 12/22/2015 4:13:32 PM MBE PREVIEW DIAGNOSTIC EXAM 71. as the prosecution alleges. His statement that he was intending to do so is circumstantial evidence that he did so. It should therefore be admissible under the present state of mind exception. (B) is incorrect. While the cousin's confrontation with the defendant may have triggered the murder, the prosecution is offering the statement in the letter for its truth (i.e., that the cousin planned to confront the defendant and subsequently did so), in large part to rebut the defendant's assertion that he had not seen the cousin in several weeks. (C) is incorrect because, as discussed above, the statement falls within the hearsay exception for statements of present state of mind. (D) is incorrect because, while the cousin's state of mind is not directly in issue, it is relevant to show that he acted in conformity with that state of mind.

A tire company is the manufacturer of, and owns the patent for, a new and unique type of truck tire. The company is the largest employer in the state in which it is located. That state enacted a law requiring all trucks using its highways to be equipped with the company's unique truck tires. An out-of-state trucking company filed suit against the state, and eventually the United States Supreme Court held that the state's statute violated the Commerce Clause. Subsequently, Congress enacted a law that required all trucks engaged in interstate commerce to use the company's unique tires. Is this federal law constitutional? (A) Yes, because Congress's control over interstate commerce is very broad. (B) Yes, only if Congress can demonstrate that the use of tires of another design somehow unduly burdens interstate commerce. (C) No, if most states permit tires of another design. (D) No, because the federal law reverses a decision of the Supreme Court, and the Supreme Court is the sole arbiter of constitutional issues.

(A) The law should be held constitutional. Congress has very broad power over interstate commerce, and it may enact laws that regulate commerce. (B) is a misstatement of the law, because Congress MPQ 100 preview diagnostic exam and divider P.indd 73 12/22/2015 4:13:32 PM 74. MBE PREVIEW DIAGNOSTIC EXAM is not required to find that there exists some burden on interstate commerce before it enacts laws regulating commerce. (C) is wrong because it is Congress that is empowered to regulate commerce between the states. Current state laws have no effect on whether Congress can, or cannot, regulate. (D) is wrong because the federal statute did not reverse the decision of the Supreme Court. It merely enacted a requirement as a matter of federal law, while the Supreme Court had ruled that a state law on the subject was invalid.

At 3 a.m. on the state turnpike, a driver was stopped for driving 30 m.p.h. over the posted speed limit. Because of the speed at which she was traveling and because the driver possessed an out-of-state driver's license, the officer decided to place the driver under arrest and take her to the station house. Under state law, such an arrest is valid. After placing the driver in his patrol car, the officer searched the passenger area of the automobile. Under the front seat he found a small package containing what he immediately determined to be marijuana. If the driver is charged with possession of marijuana, how should the court rule on her motion to suppress the marijuana? (A) Grant it, unless the officer had probable cause to believe the car contained items that were legitimate to seize. (B) Grant it, because the search was of an area outside the control of the arrestee. (C) Deny it, because the search was incident to a valid arrest. (D) Deny it, if the court determines the officer had a reasonable fear of the driver

(A) The motion to suppress will be granted unless the officer had probable cause to believe that the car contained seizeable items. The automobile exception to the warrant requirement permits a stopped vehicle to be searched in the absence of a warrant if the police have probable cause to believe that the vehicle contains seizeable evidence. (B) is incorrect. A search incident to arrest may not include the passenger compartment of a vehicle from which the person was arrested unless (i) the person has not been secured or (ii) the arresting officer has reason to believe that the vehicle contains evidence of the crime for which the arrest was made. [Arizona v. Gant (2009)] Here, the officer was able to secure the driver and the arrest was for speeding, so neither justification applies. However, even though the evidence is not admissible under the exception for a search incident to arrest, it is admissible under the automobile exception (as discussed above) if the officer had probable cause to believe that the car contained seizeable items, as stated by choice (A). (C) is incorrect because it is too broad. The passenger compartment of a vehicle cannot be searched incident to arrest unless one of the two criteria stated above are met. (D) is incorrect because "reasonable fear" is not a ground for searching a vehicle. A vehicle may be searched incident to arrest under the circumstances detailed above. It may also be searched under the MPQ 100 preview diagnostic exam and divider P.indd 82 12/22/2015 4:13:33 PM MBE PREVIEW DIAGNOSTIC EXAM 83. automobile exception if the police have probable cause to believe that the vehicle contains seizeable evidence. Finally, a vehicle can be searched without a warrant pursuant to an investigative stop (i.e., a Terry stop) if the police have reason to believe that the person stopped is armed and dangerous. But simple "fear" is not enough.

A software designer and software programmer ran into each other at a crowded software convention. The designer suggested to the programmer that they become partners. The programmer quickly retorted, "Look buddy, I like you, you're a nice guy and all that, but I'd never become your business partner, because when it comes to software design, you are totally incompetent." This remark was overheard by the private secretary to the CEO of a major software company that often subcontracted portions of jobs to smaller software design businesses, although it had never used this designer before. If the designer sues the programmer for defamation, who is likely to prevail? (A) The programmer, if her statement about the designer's competence is true. (B) The programmer, because there was no publication. (C) The designer, unless the programmer did not know her statement could have been overheard. (D) The designer, if he suffered pecuniary loss as a result of the programmer's statement.

(A) The programmer is likely to prevail if the statement is true. A defamatory statement is actionable only if it is false. The programmer has a complete defense if her statement about the designer MPQ 100 preview diagnostic exam and divider P.indd 94 12/22/2015 4:13:33 PM MBE PREVIEW DIAGNOSTIC EXAM 95. is true. (B) is incorrect. There likely was publication of the statement because it was made at a crowded party and overheard by a third party; negligence is sufficient to establish this element. (C) is incorrect for the same reason. The programmer could also be liable if she was negligent in allowing third persons to overhear her statement. (D) is incorrect because the programmer's statement disparaging the designer's competence in his profession is slander per se, and therefore actionable, even without a showing of pecuniary loss.

Two partners bought a commercial building from an owner. They paid cash for the building and took title as joint tenants with right of survivorship. Several years later, the first partner executed a mortgage on the building to secure a personal loan to a bank. The second partner had no knowledge of the mortgage to the bank. The state in which the commercial building is located recognizes the lien theory of mortgages. The first partner died before paying off his loan. He left all of his property by will to his daughter, his only heir. Who has title to the commercial building? (A) The second partner has title free and clear of the mortgage. (B) An undivided one-half is held by the second partner free and clear of the mortgage, and the other one-half is held by the daughter, subject to the mortgage. (C) An undivided one-half is held by the second partner and the other one-half by the daughter, with both halves subject to the mortgage. (D) The second partner has title to the entire property, with an undivided one-half being subject to the mortgage.

(A) The second partner has title free and clear of the mortgage. When the partners bought the property, they took title as joint tenants with right of survivorship. If the joint tenancy continued until the first partner's death, then the property would pass immediately on death to the second partner. Because the second partner did not sign the mortgage, she would not be subject to it, regardless of whether she knew about it. The key to answering this question is to know whether execution of the mortgage by the first partner caused a severance of the joint tenancy. If it did cause a severance, then the first partner's one-half would not pass to the second under right of survivorship but instead would pass to the first's estate, and thus would go to the daughter by will. Whether a mortgage creates a severance or not depends on whether the state follows the lien theory or the title theory of mortgages. Lien theory means no severance; title theory means severance. Because this is a lien theory state (majority rule on the MBE), there was no severance; thus, the joint tenancy remained intact. On the first partner's death, the joint tenancy ended and the first partner's interest instantly passed to the second partner. The first partner's estate got nothing; hence, the daughter could get nothing. (B) would be the correct answer if the execution of the mortgage had created a severance of the joint tenancy. The severance would have changed the joint tenancy to a tenancy in common. The second partner would keep her one-half, free of the mortgage she did not sign, and the daughter would inherit the first partner's one-half, subject to the mortgage the first partner signed. Because we are told this is a lien theory state, there was no severance when the mortgage was executed so the joint tenancy remained intact until the first partner's death. (C) is incorrect. Not only is the joint tenancy unsevered so that it remains intact to give title to the entire property to the second partner, but also under no circumstances could the second partner be held liable for a mortgage she did not enter into. (D) is incorrect. When the first partner died, the property passed free and clear to the surviving joint tenant. The mortgage signed by the first partner did not sever the joint tenancy because this is a lien theory state. The mortgage can be held only against the property the first partner has; when the first partner died, the right of survivorship operated to end the first partner's interest and automatically vested it in the survivor.

A husband purchased a cabin as his separate property. The husband provided in his will that the cabin be devised "to my sister-in-law for so long as my wife is alive, remainder to my son." Shortly thereafter, the husband died. When the sister-in-law learned of her interest in the cabin, she decided to lease the property. She entered into a valid written lease for a term of three years with a tenant, who agreed to pay $3,000 per year rent, with $1,500 due in advance every six months. The tenant paid the first $1,500 and took possession of the property. Two years later, one week after the tenant had paid the $1,500 due for the ensuing six-month period, the wife died. The son demanded that the tenant surrender possession of the cabin, but the tenant refused. In an appropriate action for possession of the disputed property, is the son likely to prevail? (A) Yes, because his rights are superior to the tenant's. (B) Yes, if he reimburses the tenant for the portion of the six months' rent already paid to the sister-in-law that is attributable to the period after the tenant surrenders possession. (C) No, because he did not wait to bring the action until the period for which the tenant had already paid rent had elapsed. (D) No, because the tenant has the right to remain in possession for the last year of his lease as long as he pays the final $1,500.

(A) The son will probably prevail. Under the husband's will, the sister-in-law's interest will terminate upon the wife's death. Regardless of whether the court determines it to be a life estate per autre vie (i.e., a life estate with another's life as the measuring life) or a fee simple determinable (the wife's death being the terminating event), the sister-in-law's interest will end when the wife dies and the property will go automatically to the son. Because the sister-in-law's estate will end on the wife's death, she cannot affect any rights of any holder of a future interest that will become possessory on that event. Any lease purporting to continue after the wife's death is simply invalid as of that event. When the wife dies, the son has a fee simple and the tenant has nothing. Thus (A) is correct. (B) is incorrect. The son received a fee simple on the wife's death, and he is not bound by any actions of the holder of the life estate purporting to lease the property for any period that MPQ 100 preview diagnostic exam and divider P.indd 78 12/22/2015 4:13:33 PM MBE PREVIEW DIAGNOSTIC EXAM 79. would extend beyond the terminating event. (C) is incorrect because the son's right to possession begins on the instant of the wife's death. The life tenant can do nothing to extend that date. (D) is incorrect. The tenant's right to possession ends on the wife's death, regardless of what the tenant's lease from the sister-in-law may say. She could give no more than she had, and her interest ended on the wife's death

After the failure of a state bill granting gay and lesbian couples the right to marry, 30 students from a local university marched on the state capitol to protest, carrying signs with slogans such as "let gays marry" and "no religious tyranny." As they marched, about 15 people began following them, shouting anti-gay remarks. At the capitol building, they were met by 50 officers in full riot gear. The leader of the students addressed his followers in the park across the street, vigorously denouncing the legislature's actions, which caused the counterprotestors to become more vocal. The officer in charge told the leader that he must end his speech because a riot was about to start. The leader refused and was arrested and convicted of disorderly conduct. If the leader appeals his conviction on constitutional grounds, will the conviction be reversed? (A) Yes, because the leader's arrest constituted an interference with his First Amendment right to free speech. (B) Yes, because the leader's arrest constituted undue interference with the students' right to peaceably assemble under the First Amendment. (C) No, because the leader's speech caused an immediate and substantial threat to public order. (D) No, because the leader had a duty to obey the police officer since other avenues of communication likely were available.

(A) The speaker's conviction will be reversed. A park is a public forum. The government can limit rights of speech in such a forum only when there is a serious and imminent threat to the public order. It can restrict the speech of a speaker because of an unruly audience only in the rare case when the police are absolutely unable to control the crowd. [See Feiner v. New York (1951)] In this question, the conditions under which the police can prevent a speaker from continuing because of an unruly crowd have not been met. There were 50 police officers who would have been able to restrain or subdue anyone who appeared to be intent on committing violence. Hence, (C) is incorrect. (B) is incorrect. No one in the audience has raised any constitutional argument. The speaker probably cannot raise the audience members' First Amendment rights in this situation. (D) is incorrect. The state may not limit access to a public forum on the sole basis that there are other times and places where the right of free speech can be exercised. The state must show a more substantial reason.

A homeowner agreed to sell his home to an accountant. He gave the accountant a general warranty deed and the accountant gave him $86,000, his asking price. The deed was recorded. A few years later, the accountant sold the property to a doctor, who paid her $125,000 for the property. To save on attorney's fees, the accountant went to her local office supply store to purchase a general warranty deed form. The store was out of those forms, but the clerk suggested that she use the form labeled "Quitclaim Deed," asserting that it would transfer the title just as well as the other form. The accountant purchased the form and filled in the blanks with the appropriate information she copied from her old deed. The doctor did not hire a lawyer to represent him in the purchase of the house. The doctor accepted the deed from the accountant and gave her $125,000. Soon after the doctor moved into the house, it was discovered that the homeowner's title was not good. The true owner now demands that the doctor vacate. Title is judicially determined to be with the true owner, and the doctor is forced out. Does the doctor have any action against the homeowner or the accountant based on any covenant for title? (A) Yes, the doctor can sue both and can recover $125,000. (B) Yes, the doctor can sue the homeowner, but not the accountant, and can recover his full $125,000. (C) Yes, the doctor can sue the homeowner, but not the accountant, and can recover only $86,000. (D) No, the doctor can sue neither the homeowner nor the accountant.

(C) The doctor can sue the homeowner but can recover only $86,000. The homeowner gave a warranty of title to the accountant when he gave her a general warranty deed. This warranty runs with the land and can be enforced by any subsequent purchaser. Damages are limited, however, to the purchase price received by the warrantor. Because the homeowner received only $86,000 from the accountant, that is all that the doctor can recover. The doctor gets nothing from the accountant because she gave only a quitclaim deed, which gives no warranty of title. (A) is incorrect because the doctor cannot prevail over the accountant at all, and he can recover only $86,000 against the homeowner. (B) is incorrect. While the doctor can win over the homeowner and not the accountant, he is limited to the money the homeowner received from the accountant—$86,000. (D) is incorrect. Because the homeowner gave a general warranty deed, that warranty for title runs with the land and can be enforced by any subsequent purchaser. There are two (sometimes three) future covenants contained in a general warranty deed—the covenants of quiet enjoyment, MPQ 100 preview diagnostic exam and divider P.indd 68 12/22/2015 4:13:32 PM MBE PREVIEW DIAGNOSTIC EXAM 69. warranty, and (sometimes) further assurances. Quiet enjoyment is a covenant that the grantee will not be disturbed in her possession or enjoyment of the property by a third party's lawful claim of title. Warranty is a covenant that the grantor agrees to defend on behalf of the grantee any lawful or reasonable claims of title by a third party. Further assurances is a covenant to perform whatever acts are reasonably necessary to perfect the title conveyed if it turns out to be imperfect. These three covenants are future covenants that are breached, if at all, only on interference with the possession of the grantee or her successors; thus, they "run" with the grantee's estate. Here, the future covenants contained in the accountant's deed from the homeowner run with the land, allowing the doctor to seek recourse against the homeowner, the original grantor, for the disturbance in possession.

The federal government has complete jurisdiction over certain parkland located within a state. To conserve the wildlife inhabiting that land, the federal government enacted a statute forbidding all hunting of animals in the federal park. That statute also forbids the hunting of animals that have left the federal park and have entered the state. A hunter has a state hunting license, authorizing him to hunt deer anywhere in the state. On land within the state located adjacent to the federal park, the hunter shot a deer he knew had recently left the federal land. If the hunter is prosecuted for violating the federal hunting law, what is the strongest ground supporting the constitutionality of the federal law forbidding the hunting of wild animals that wander off of federal property? (A) This law is a necessary and proper means of protecting United States property. (B) The animals are moving in the stream of interstate commerce. (C) The police powers of the federal government encompass protection of wild animals. (D) Shooting wild animals is a privilege, not a right.

(A) The strongest ground for supporting this provision is the Necessary and Proper Clause. Congress has the authority to legislate to protect federal parklands. The federal law safeguards wild animals that wander off federal parklands. The Necessary and Proper Clause allows Congress to choose any means to carry out its constitutional powers. Thus, it may regulate hunting to protect the federal interest in safeguarding the wild animals. (B) is wrong because it rests on an unjustified factual statement, that the "animals are moving in the stream of interstate commerce." There are no facts describing the movement patterns of the animals. (C) is wrong because there is no federal police power to protect wild animals. Generally, the federal government lacks the police power; the police power belongs to state and local governments. Although Congress possesses the police power in legislating for federal lands, this choice is not limited to those circumstances. It speaks broadly of a federal police power to protect wild animals, and none exists. (D) is wrong because the Supreme Court has discarded the distinction between rights and privileges as a constitutional principle. Moreover, it is a distinction used to determine when the government must provide procedural due process before deprivations of property or liberty. This question does not involve the Due Process Clause.

The plaintiff, who resides in the Southern District of State A, was involved in a three-car accident in the Northern District of State A. The plaintiff intends to file a negligence action against the other two drivers in federal district court. One defendant resides in the District of State B and the other resides in the District of State C. In which federal district(s) is venue proper? (A) The Northern District of State A only. (B) The District of State B and the District of State C. (C) The Northern District of State A, the District of State B, and the District of State C. (D) The Northern District of State A, the Southern District of State A, the District of State B, and the District of State C.

(A) Venue is proper only in the Northern District of State A. Federal venue in diversity actions is proper in (i) the district in which any defendant resides if all defendants reside in the same state; and (ii) the district in which a substantial part of the events or omissions giving rise to the claim occurred. Here, the accident occurred in the Northern District of State A, making that district a proper venue under prong (ii). However, given that the defendants here reside in different states, venue cannot be based on the residence of the defendants. Thus, choices (B), (C), and (D) are incorrect. Choice (D) is also incorrect because venue is not based on the residence of the plaintiff.

A farmer fleeing from a bull that had gotten out of its pen bolted for the nearest safety—the fence between the pen and a melon grower's melon patch. Leaping over, the farmer landed on and shattered several melons. Hearing the melee, the melon grower ran from a hiding place where he had been waiting to try to catch some children who had been stealing melons. He pointed his shotgun, which was loaded with rock salt, and exclaimed, "Stop or I'll shoot." Just then two juvenile melon thieves decided to make their escape and ran from where they had been crouching. The melon grower aimed his shotgun at them and prepared to fire but slipped, and the load of rock salt struck the farmer, whom the melon grower had recognized and had not intended to hit. Has the melon grower committed an assault against the farmer? (A) Yes, because he actually pointed the shotgun at the farmer. (B) Yes, if the farmer reasonably believed that the melon grower might shoot him. (C) Yes, because assault is included within the tort of battery, and the melon grower committed a battery when he shot the farmer in the shoulder. (D) No, because the shotgun was only loaded with rock salt.

(B) An assault is committed when the victim is put in reasonable apprehension of an offensive touching. If the farmer believed that the melon grower might shoot him, and the belief was objectively reasonable, then the melon grower's action amounts to assault. The conditional threat, "stop or I'll shoot," is sufficient for an assault. Because the melon grower was pointing a rifle at the farmer at the time the threat was made, the requirement of immediacy was fulfilled. The melon grower cannot claim defense of property because the farmer had a privilege based on private necessity to enter the melon grower's land. (B) is a better answer than (A), because the melon grower would only have committed an assault if the farmer had a reasonable apprehension of a harmful or offensive contact. (C) is a misstatement of the law. Assault is not a lesser included tort MPQ 100 preview diagnostic exam and divider P.indd 67 12/22/2015 4:13:32 PM 68. MBE PREVIEW DIAGNOSTIC EXAM within battery. (D) is incorrect because what is contained in the shotgun is irrelevant. The gun created apprehension of a harmful or offensive contact.

Two police officers stopped an automobile for improperly proceeding through a red light. When one of the officers approached the car, he observed some hand-rolled cigarettes on the dashboard. He ordered the driver out of the car, examined the cigarettes, and determined that they were joints of marijuana. The officer then arrested the driver and guarded him in the back seat of the police cruiser while the second officer searched the entire car, including the trunk. In the trunk, he found two rare paintings that had recently been stolen from the city's art museum. The driver was charged with possession of stolen goods and brought a motion to suppress the introduction of the paintings into evidence. If the driver's motion is denied, it will be because the second officer conducted which type of search properly? (A) Search incident to an arrest. (B) Automobile search. (C) Custody search. (D) Inventory search.

(B) Because of a lessened expectation of privacy in a car (as compared to a home) and because of the inherent mobility of a car which can prevent the police from easily obtaining a search warrant and searching it, the police are permitted to make a complete search of an automobile if there is probable cause to believe that the car contains the fruits, evidence, or instrumentalities of a crime. [Carroll v. United States (1925)] When the police have probable cause to search an entire vehicle, they may conduct a warrantless search of every part of the vehicle and its contents. [United States v. Ross (1982)] While it is doubtful that the marijuana joints provided sufficient probable cause to search the entire vehicle, this is the best reason to deny the motion, because all of the other choices are clearly incorrect. (A) is incorrect because a lawful search of a vehicle incident to an arrest includes a search of the defendant and, in limited circumstances, the area of the vehicle within the defendant's control. [Arizona v. Gant (2009)] In this case, as the trunk is outside the defendant's area of control, the search went beyond the scope of a proper search incident to an arrest. Therefore, this was an invalid search incident to arrest and the driver's motion would be granted on this basis. (C) is incorrect. Technically, there is no such thing as a custody search. "Custody" here may refer to custody of the person, in which case it is a search incident to arrest discussed above, or custody of the vehicle, which is an inventory search. An inventory search occurs when the officers properly impound a car and search it while it is in their control to make sure that the assets of the owner of the car are fully accounted for. [South Dakota v. Opperman (1976)] This choice is incorrect because it does not articulate a legal basis to conduct the search, as the car was not impounded. (D) is incorrect. In this case, the police did not take possession of the car, so this is not the best reason to deny the motion.

A consumer purchased an air compressor, designed for powering tools and inflating tires, to use in a remodeling project. The consumer discovered that the reservoir and nozzle for a paint compressor gun he owned fit the threads on the nozzle of his air compressor. He decided to use the manufacturer's compressor rather than buying the compressor that went with the spray gun because he needed to paint a fence. After painting for a few minutes, the paint reservoir on the spray gun exploded, causing severe injuries to the consumer. The consumer subsequently brought a strict products liability action against the air compressor manufacturer to recover for his damages. Evidence at trial revealed the following: (i) The spray gun reservoir had exploded because the air compressor developed too much air pressure for use in a spray device; it was for this reason that the manufacturer did not manufacture a spray painting attachment for its compressor. (ii) The manufacturer purposefully chose an unusual threading for its compressor nozzle to prevent its use with most spray painting systems, but the spray gun that the consumer had was a foreign-manufactured model that happened to use the same threading. (iii) When used with the tools and attachments produced by the air compressor manufacturer, the air compressor was completely safe and suffered from no design or manufacturing defects. If the jury finds for the manufacturer, what is the most likely explanation? (A) The manufacturer exercised due care in the design and manufacture of the paint sprayer. (B) It was not foreseeable that the compressor would be used by consumers to attach power spray devices. (C) The compressor suffered from no design or manufacturing defects when used in the intended manner. (D) The consumer misused the compressor when he attached it to his spray-painting gun.

(B) If the jury finds for the manufacturer, it will be because the misuse by the consumer was not foreseeable. A manufacturer is liable for a defective product, even if the plaintiff misuses it, as long as that misuse is foreseeable. Foreseeability in this case is an issue for the trier of fact. If the jury finds that the misuse was not foreseeable, the manufacturer will prevail. (A) is incorrect because the consumer's action is based on strict liability. (C) and (D) appear to be true based on the facts, but they ignore the key issue that the jury must decide, which is whether the consumer's misuse was foreseeable.

A state operates a nuclear power plant on a river. The plant uses river water for cooling and discharges water back into the river 10 degrees warmer than when it was taken in. While the discharged water quickly cools, it has adversely affected the business of a downstream ice cutting operator, located in a neighboring state. The ice cutter successfully urged his state to bring suit in the United States Supreme Court against the state operating the power plant, alleging damage to its environment and seeking an injunction against the thermal discharge. What should the United States Supreme Court do? (A) Dismiss the action, because the suit is really one by the ice cutter against the state operating the power plant and is barred by the Eleventh Amendment. (B) Hear the matter on the merits, because the ice cutter's state is suing in its own right and jurisdiction is proper. (C) Dismiss the action, because it does not have original jurisdiction. (D) Remand the case to the district court for trial, because it does not have the time to function as a trial court.

(B) The Court should hear the case on the merits. A state can sue another state to protect its natural resources for the benefit of its own citizens. [Pennsylvania v. West Virginia (1923)] The ice cutter's state is suing the state operating the power plant under this doctrine. Under Article III, the Supreme Court has original and exclusive jurisdiction over controversies between two states. Because this is a controversy between two states, Supreme Court jurisdiction is proper. (A) is incorrect. The ice cutter's state is a real party in interest, and the Eleventh Amendment, which bars suits by citizens of one state against another state in federal court, is inapplicable. (C) is incorrect. Article III, Section 2(2) of the Constitution confers exclusive original jurisdiction to the Supreme Court over all controversies between two or more states. (D) is incorrect. Article III, Section 2 of the Constitution confers exclusive original jurisdiction on the Supreme Court in cases between two or more states. In such cases, the Court must serve as a trial court and even Congress may not limit that jurisdiction. [See Marbury v. Madison (1803)]

A mother had her lawyer prepare and execute two notarized deeds: one conveying a 640-acre farm to her daughter and the other conveying a 590-acre farm to her son. The mother produced the deed to the daughter, who told her mother to keep the deed just in case the daughter misplaced it. The mother held on to the deed. The next day, the mother went to her lawyer's office to have the deeds recorded, but she forgot the daughter's deed at home. The mother gave the lawyer the son's deed and promised to bring the daughter's deed in the next day. Later that afternoon, the lawyer recorded the son's deed. That night, the mother suffered a stroke and died in her sleep. The daughter's deed was never recorded but was found in the mother's home after her death. In her will, the mother left all of her property to her daughter, her son, and the local animal shelter, in three equal shares. Aside from her modest home and some small bank accounts, the mother had no appreciable estate. If the animal shelter challenges the deeds to the farms on grounds of lack of proper delivery, what is the likely result? (A) The shelter will win on both farms. (B) The shelter will lose on both farms. (C) The shelter will win on the 640-acre farm but lose on the 590-acre farm. (D) The shelter will win on the 590-acre farm but lose on the 640-acre farm

(B) The animal shelter will lose on both farms. As to the deed to the son conveying the 590-acre farm, delivery is presumed because the deed was validly recorded, even though the son did not physically receive it and does not seem to have known of its existence. As to the deed to the daughter conveying the 640-acre farm, the fact that she did not physically keep the deed when it was presented to her does not show lack of delivery under the circumstances. The key to delivery is the grantor's intent to pass all legal controls. Because the daughter simply asked her mother to safeguard the deed, we can presume delivery under these circumstances. But because the deed to the daughter was never recorded, we cannot rely on the rule that provided valid delivery in the son's case. (A) is incorrect. Both deliveries were valid. The son's deed was valid by reliance on the presumption that recordation shows a valid delivery, and the daughter's deed on the circumstances of the gift that show proper intent. (C) is incorrect. The animal shelter does lose on the 590-acre farm, but does not win on the 640-acre farm because the daughter's having her mother keep the deed does not show lack of delivery. (D) is incorrect. The animal shelter does lose on the 640-acre farm, but does not win on the 590-acre farm because the recordation of the son's deed gives a presumption of valid delivery

For many years, a farmer has grown and harvested a sizeable wheat crop on his land. On May 1, the farmer entered into an agreement with a bakery to supply the bakery with 10,000 bushels of wheat at $15 a bushel. The agreement called for the farmer to deliver the bushels to the bakery's plant on or before September 1. In mid-July, it became apparent that the year's wheat crop grown in the United States was going to be substantially smaller than originally anticipated. As a result, the price of wheat on August 1 rose to $35 a bushel. On August 10, the farmer informed the bakery that he would be unable to deliver any bushels to the bakery on September 1 because he had sold his entire harvested crop to other consumers. If the bakery sues the farmer for breach of contract, will the bakery prevail? (A) Yes, as long as it files suit on or after September 1. (B) Yes, regardless of whether it sues before or after September 1. (C) No, because the sharp increase in the price results in a commercial frustration of the original deal, which excuses the farmer's performance. (D) No, because the unforeseen rise in price is a substantial change of circumstances that excuses both parties.

(B) The bakery will prevail regardless of when it sues. The farmer's statement of August 10 that he would be unable to deliver because he had sold all of his crop to others constitutes a repudiation of the contract, which entitles the bakery to sue at once rather than wait for the September 1 performance date. [See U.C.C. §2-610] The bakery will prevail unless there have been some unanticipated circumstances that would excuse performance, but a rise in price due to poor crop performance is not sufficient to excuse performance under the doctrine of commercial impracticality. The farmer appears to have repudiated merely to make a greater profit. (A) is incorrect—because the farmer repudiated, the law does not require the bakery to wait until the due date for performance to arrive. It may sue at once. [See U.C.C. §2-610] (C) is incorrect because the facts do not indicate that the farmer has been harmed at all by the price increase, or that he cannot perform the contract with the bakery. As a matter of fact, he is simply attempting to gain a windfall by repudiating the contract with the bakery to sell at a higher price. (D) is incorrect because the facts do not indicate that the circumstances that led to the poor crop were unforeseen. Crop performance is risky and variable, and a poor crop can certainly result in a price increase MPQ 100 preview diagnostic exam and divider P.indd 64 12/22/2015 4:13:32 PM MBE PREVIEW DIAGNOSTIC EXAM 65. by virtue of the operation of the law of supply and demand. The farmer must be assumed to have anticipated such possibilities and taken such risks.

On January 1, state legislation was enacted that prohibited private employers, under penalty of civil fines, from hiring any person who had not been a resident of that state for 10 years. The legislation also required private employers to immediately discharge any employee who had not resided in the state for a minimum of 10 years. The new statutes additionally provided that the applicable civil fines would be assessed by the state department of employment inspectors, at their sole discretion, any time they believed that an employer had hired or failed to discharge a person not meeting the residency requirements, and that the civil fines would be imposed on any employer who had hired or failed to discharge such persons on or after January 1. An employee within the state, who is a nonresident, brought suit challenging the state statute. Which of the following arguments would most likely result in a favorable ruling for the challenger? (A) The statute is an unconstitutional Bill of Attainder. (B) The statute violates the Privileges and Immunities Clause of Article IV. (C) The statute violates the Equal Protection Clause of the Fourteenth Amendment. (D) The statute violates the Ex Post Facto Clause.

(B) The best argument is based on the Privileges and Immunities Clause. State laws that discriminate against out-of-state residents in terms of the right to earn a living trigger the Privileges and Immunities Clause of Article IV and are almost always unconstitutional. (A) is incorrect. A Bill of Attainder requires, inter alia, a legislative act which is intended to "punish" an identifiable group. Punishment is an unlikely motive of the statute. (C) is incorrect. Out-of-state residents are not considered a protected class under the Fourteenth Amendment Equal Protection Clause. This would more than likely be a Fourteenth Amendment Due Process Clause violation because the government may not take a recognized "liberty" or "property" interest of a person without some procedural due process. (D) is incorrect. Only retroactive criminal laws can violate the ex post facto clause.

Late one night, a young couple were killed instantly when their car was struck by a speeding truck as the couple's car crossed an intersection with the light green in their favor. Several weeks later, a burglar awaiting trial on burglary charges asked a jail officer to let him speak with a highway patrol officer. When the highway patrol officer came to the cell, the burglar told him that he was the driver of the truck that had struck the car and had been speeding away from a burglary when the accident occurred. The burglar was charged with felony murder, on the theory that he had not yet reached a place of temporary safety when the accident occurred. At trial, the prosecution seeks to introduce the burglar's statements to the highway patrol officer regarding the events of the night of the accident. The burglar's attorney objects. Which of the following is the strongest argument for permitting the statements into evidence? (A) The burglar had not been charged in connection with the auto accident at the time the statements were made to the highway patrol officer. (B) The burglar made the statements spontaneously, without inducement or interrogation by the police. (C) The highway patrol officer had no connection with the burglary investigation for which the burglar had been incarcerated. (D) The burglar's statements were not the product of coercion by the police officers.

(B) The best argument is that the burglar made the statements spontaneously. Prior to a suspect's being charged with a crime, the Fifth Amendment privilege against compelled self-incrimination is the usual basis for ruling on the admissibility of a confession. [Miranda v. Arizona (1966)] Under Miranda, statements made during custodial interrogations are inadmissible unless the defendant is first warned of his right to remain silent and his right to an attorney. Thus, Miranda applies only when the defendant is in custody and only when the defendant's statements are the result of interrogation. Although almost any words or actions on the part of the police that they should know are reasonably likely to elicit an incriminating response qualify as interrogation, Miranda does not apply to spontaneous statements not made in response to interrogation. Here, the police did nothing to solicit the statement from the burglar; it was spontaneous. Thus, (B) is correct. (A) is incorrect because the defendant need not yet be charged for Miranda rights to apply as long as he is in custody (i.e., not free to leave). Being in jail on another charge (as the burglar was) satisfies the custody requirement. (C) is incorrect because the fact that the officer who took the burglar's admission had nothing to do with the investigation of the burglary does not alter the rules of Miranda—questioning that is totally unrelated to the matter for which the accused is in custody may still violate the accused's Miranda rights. (D) is incorrect. Due process requires that a confession be voluntary (i.e., not the product of police coercion). The Miranda rule, however, goes beyond voluntariness. It makes inadmissible all statements obtained without Miranda warnings or without a valid waiver of Miranda rights, not just statements actually coerced by the police.

A defendant calls a secretary present at a meeting between the defendant and the plaintiff in which attempts were made to negotiate a settlement. The secretary offers to testify that the plaintiff said at the meeting, "Well, maybe it wasn't fraud—I'll settle for refund of the purchase price plus $50,000." The plaintiff objects. Is the offered testimony admissible? (A) Yes, as a statement by a party-opponent. (B) Yes, because statements made in the course of settlement negotiations are admissible where relevant, even though offers to compromise are excluded. (C) No, because of public policy. (D) No, because the statement was made in the office of the defendant, an attorney, and hence privileged.

(C) The testimony is inadmissible on public policy grounds. This question involves the admissibility of settlement offers ("I'll settle for a refund of the purchase price plus $50,000") and statements made in the context of settlement negotiations ("Well, maybe it wasn't fraud"). Both settlement MPQ 100 preview diagnostic exam and divider P.indd 65 12/22/2015 4:13:32 PM 66. MBE PREVIEW DIAGNOSTIC EXAM offers and statements made during settlement negotiations are inadmissible. [Fed. R. Evid. 408] This is because public policy favors the voluntary settlement of disputes. If a settlement offer has been made in a case that ultimately goes to trial, admission of the offer into evidence would probably prejudice the party who made the offer. Thus, if such evidence were admissible, there would be a disincentive to parties making settlement offers. Similarly, if statements made during settlement negotiations could be admitted into evidence in cases that ultimately go to trial, then that would be a disincentive to enter into settlement negotiations or to discuss settlement in a freewheeling way. In either event, the public policy favoring settlements bars use of such statements. (A), which asserts the offered testimony is admissible as a statement by a party-opponent (to show plaintiff's recognition of the weakness of his case), is plainly incorrect. Because of the public policy valuing settlements, this type of testimony would not be admissible. (B) is incorrect because it is only half true. Offers to compromise are excluded, as (B) says, but (B) is incorrect in asserting that statements made in the course of settlement negotiations are admissible where relevant. (D) is incorrect. The offered testimony involves a statement made by the plaintiff to the defendant, not a statement made by a party to the party's attorney. Thus, the attorney-client privilege is not involved. Similarly, the attorney work product privilege is inapplicable. A settlement offer made by a party or an opposing party is not attorney work product. (D)'s assertion—that any statement made in the office of an attorney is privileged—is obviously incorrect.

On January 30, a company that designs and builds generators to standard industrial specifications received a telephone call from a buyer who ordered two generators at a price of $25,000 each. The parties agreed that delivery of the first generator would be on March 15, and the second on April 30. Payment was to be made no more than 30 days after delivery. On March 12, the company delivered the first generator, which the buyer accepted. On April 22, the company completed the second generator but had not yet notified the buyer. On April 23, the buyer, having made no payment to the company, canceled the agreement. The company brings an action against the buyer for breach of contract. How much should the company recover in damages? (A) Nothing. (B) $25,000 only. (C) Damages for total breach of contract for the sale of two generators, because the buyer accepted part performance. (D) Damages for total breach of contract for the sale of two generators, because the goods were made for the buyer.

(B) The company should recover $25,000 only. Contracts for goods for $500 or more must be evidenced by a writing to be enforceable. There are three exceptions to this rule: specially manufactured goods unsuitable for resale in the seller's regular course of business, contracts admitted in court, and contracts partially accepted (enforceable to the extent of the acceptance). Here, the contract was for $50,000 and was oral. Thus, it will be enforceable only if one of the exceptions applies. The buyer's acceptance of the first generator constitutes part acceptance that will make the buyer liable to the extent of the acceptance: $25,000. Therefore, (B) is correct and (A) is incorrect. (C) is incorrect because partial acceptance renders the buyer liable only for the part accepted, not the entire contract. (D) is incorrect because, while the goods were made for the buyer, they were suitable for resale in the company's business, because they were built to standard industry specifications.

A homebuyer filed a breach of contract action in a State A state court against the contractor (a corporation) that agreed to build the home and sell it to the buyer. The homebuyer's complaint joined as an additional defendant the contractor's agent who negotiated the contract and signed it on behalf of the contractor. The action sought $150,000 in damages from both defendants. The contractor is a citizen of State B. The homebuyer and the contractor's agent are citizens of State A. Six months after filing the action, the homebuyer dismissed the claims against the agent, leaving only the claims against the contractor. The corporation immediately and appropriately filed a notice of removal. May the contractor remove the action to federal district court? (A) Yes, because the action can be properly removed at any time after the action was filed. (B) Yes, because, while the action was not properly removable when it was filed, it now can be properly removed to federal district court. (C) No, because, while the requirements for diversity of citizenship jurisdiction now are satisfied, the time in which removal is allowed has passed. (D) No, because removal is allowed only if federal subject matter jurisdiction exists at the time the complaint is filed.

(B) The contractor may remove the action. A defendant may remove an action that could have originally been brought in the federal courts (because subject matter jurisdiction based on either a federal question or on diversity of citizenship was present). Diversity of citizenship jurisdiction is available when (i) there is complete diversity of citizenship, meaning that each plaintiff is a citizen of a different state from every defendant; and (ii) the amount in controversy exceeds $75,000. Generally speaking, a defendant has 30 days from the date he receives the initial summons or complaint to remove a case. However, if a case becomes removable on the basis of diversity at a later date, he has 30 days to remove the case from the date the defendant is served with the document that first makes the case removable. That said, for cases based on diversity, removal may not take place more than one year after the case was filed. In the instant case, the homebuyer (from State A) initially sued the agent (also from State A) and the contractor (from State B) for $150,000. Thus, the case was not initially removable. The case then became removable when the homebuyer dismissed the agent from the case, leaving the State B corporation as the sole defendant. At this point, the corporation has 30 days to remove the case, and the one-year restriction does not come into play because the facts state that only six months have passed since the case was filed. As a result, (B) is the correct answer. (A) is incorrect because the case was not initially MPQ 100 preview diagnostic exam and divider P.indd 66 12/22/2015 4:13:32 PM MBE PREVIEW DIAGNOSTIC EXAM 67. removable. The claim was for a state law breach of contract, so no federal question was presented, and complete diversity was initially lacking, so diversity of citizenship jurisdiction was not available. (C) is incorrect because the 30-day period has been met (since the corporation immediately filed a notice of removal), and only six months have passed since the case was filed in state court, making the case removable on the basis of diversity of citizenship jurisdiction. (D) is an incorrect statement of the law. The grounds for removal need not exist at the time the case is filed. If grounds for removal come up later, the case may still be removed, subject to certain restrictions.

A biker sued a driver for injuries allegedly sustained in an automobile accident. The driver's defense was that the biker was the cause of the accident, because the biker ran a red light and crashed into the driver's car. The driver's principal witness testified that he was standing on the corner when the accident occurred, and he observed the biker run the red light and crash into the driver's car. On cross-examination of the witness, the biker's lawyer asked, "Isn't it true that on the day of the accident you were 300 miles away from the scene attempting to extract protection money from the owner of a seaside restaurant?" The driver's attorney objected to the question, but the objection was overruled by the court. The witness then refused to answer the question, invoking the privilege against selfincrimination. The biker's attorney moved to have the witness's direct testimony stricken from the record. How should the court rule on the motion? (A) Grant the motion, because the privilege against self-incrimination is not applicable to civil cases. (B) Grant the motion, because the biker has not had an adequate opportunity for meaningful cross-examination. (C) Deny the motion, because the witness has a right to refuse to answer. (D) Deny the motion, because the jury can assess the weight to be given to the witness's testimony.

(B) The court should grant the motion. This question involves the interrelationship between the exercise of testimonial privilege (the privilege against self-incrimination) and the right to crossexamine a witness claiming such a privilege. Should the witness's direct testimony be stricken because he refused to answer a question on cross-examination? The answer is yes, at least in the context of this case. Here, the question on cross-examination ("Isn't it true that on the day of the accident you were 300 miles away from the scene attempting to extract protection money from the owner of a seaside restaurant?") is centrally relevant to the witness's direct testimony (pertaining to his supposed observation of the accident). Every party has a right to adequate crossexamination of a witness who has testified for an opposing party. If adequate cross-examination cannot be obtained, the remedy is to strike the direct testimony of the witness. For this reason, (B) is correct, and (D) is an incorrect answer. (C) is incorrect, although it correctly states that the witness has a right to refuse to answer. The allegation of criminal activity contained in the question he declined to answer is very clear. There is thus no doubt that it is proper for the witness to "take the Fifth Amendment" in response to the question. However, (C) is incorrect in its conclusion, that the request to strike the witness's direct testimony should be denied. It does not logically follow that, because a witness has a right to refrain from answering a question, he has a corresponding right to have his direct testimony admitted into evidence. No such "right" exists, particularly where the invocation of a testimonial privilege has the effect of depriving a party of the right to adequately cross-examine the witness claiming the privilege. (A) is incorrect because the privilege against self-incrimination is applicable in any setting (civil case, legislative hearing, custodial interrogation of a suspect, etc.). As long as the information requested of a person could then be used against him in a subsequent criminal proceeding, that person has a right to invoke the privilege against self-incrimination.

Thirty years ago, a telephone company purchased an easement from an owner to install, inspect, repair, replace, and maintain underground transmission cables within a specified portion of the owner's property. The deed granting the easement was validly recorded, and the telephone company shortly thereafter laid underground long distance lines traversing the owner's land beneath the area described in the easement deed. Fifteen years later, the owner sold a 50-acre portion of his land to a buyer. This portion included part of the easement granted to the telephone company. The buyer erected a residence on the purchased land and planted 49 acres of walnut trees. He granted an easement to the telephone company to string aerial telephone wires. The underground easement was never disclosed. Last year, the telephone company informed the buyer that it would have to excavate underground cables and replace them with new ones. This was the first the buyer knew of the cables lying under his land. The cables, lying within the easement, are directly beneath a row of black walnut trees running the length of the buyer's property. Black walnuts have become extremely valuable, and the buyer estimates that their value is approximately $400,000. If the buyer seeks an injunction prohibiting the telephone company from undertaking the planned excavation, for which party should the trial court rule? (A) The telephone company, because the benefits of installing the new cables outweigh the harm done to the buyer. (B) The telephone company, because ownership of the easement permits it to make the excavation. (C) The buyer, because the telephone company concealed the existence of the underground easement from the buyer when it acquired the overhead easement from him. (D) The buyer, because the deed by which he purchased his property made no mention of the easement, and he had no actual knowledge of it.

(B) The court should rule for the telephone company. This question turns simply on whether or not there was a valid easement. The easement granted by the owner to the telephone company 30 years ago was properly recorded. That alone would put subsequent purchasers of the property on constructive notice of the easement. The fact that the easement was not in the deed from the owner to the buyer is irrelevant. A valid easement passes with title whether or not it is mentioned in the conveyance. For the subsequent purchaser to avoid the easement, that person must have taken without notice of it. Although there was no actual notice of the easement, a proper title search would have disclosed the easement, and the buyer is bound as if he knew. (A) is incorrect. If an easement is valid, there is no need to balance the hardships in order to determine whether the holder of the easement may make proper use of it. An easement to lay cables underground reasonably carries with it the right to inspect those cables or make replacements as is necessary. Hardship to the holder of the servient estate is not relevant if the easement is proper and the excavation is considered reasonable use of that easement. (C) is incorrect because the telephone company was under no obligation to mention its underground cable easement, and its failure to do so is meaningless. (D) is incorrect. Valid easements are conveyed along with the servient estate, regardless of whether they are mentioned in the conveyance or not. Because the easement was recorded, a title search would have disclosed it, and thus the buyer will be held to have constructive knowledge of it.

Under a program created by Congress, the federal government allocated $5 million to one of the nation's leading energy producing states "to be used solely for the development of synthetic materials to replace petroleum." The state accepted the funds and distributed $3 million from those funds to a state university, an agency of the state government. The university applied the $3 million to its ongoing energy research program, which included projects investigating the creation of synthetic replacements for petroleum products, solar power, geothermal power, biomass, and fusion power generation. The Justice Department brings suit in federal district court seeking to enjoin the expenditure of the funds in the university research programs. How should the court rule? (A) The university expenditure is unconstitutional because Congress has improperly delegated its power to spend for the general welfare to a state government. (B) The expenditure is unconstitutional as a violation of the Supremacy Clause. (C) The expenditure is constitutional because it conforms substantially with the purpose and goals of the federal program. (D) The expenditure is constitutional because the federal government's attempt to control the actions of a state agency violates the concepts of federalism.

(B) The court should rule that the expenditure is unconstitutional. The federal power to tax and spend includes the power to attach conditions to the expenditures, and to the extent the university research program conflicts with the federal legislation, the state action is superseded by the federal statute. Here, because the federal program expressly limited use of the distributed funds to development of synthetic materials to replace petroleum, the university projects involving solar, geothermal, biomass, and fusion energy would be in conflict with the statutory prescription. "Substantial" conformity is not enough, so (C) is incorrect. (D) is an incorrect statement of the law. The supremacy doctrine permits Congress to effectively control the actions of a state in spending money provided by the federal government. (A) is incorrect. There is nothing unconstitutional about the means by which the funds are disbursed.

During the nighttime, a woman broke into the house of the victim with the intention of stealing his diamond ring. When she could not find the diamond ring, she became angry, lit a match to a newspaper and threw it on the victim's bed, setting the mattress on fire. The flames destroyed the bed and a portion of the floor under the bed. In a common law jurisdiction, of which crime or crimes is the woman guilty? (A) Burglary only. (B) Arson only. (C) Burglary and attempted arson. (D) Burglary and arson.

(D) The woman is guilty of arson and burglary. She is guilty of arson because she deliberately set a fire that, in addition to burning the mattress, also burned part of the dwelling house of another, namely the floor. She is also guilty of burglary because she broke and entered into the dwelling house of another during the nighttime to commit a felony. The fact that she was not successful in committing the crime she intended or that she in fact committed another felony is irrelevant to her guilt for burglary; it is the intent to commit a felony at the time of the breaking and entering which is critical. (A) is incorrect. This choice correctly states that the woman is guilty of burglary, but she is also guilty of arson; she deliberately set a fire that, in addition to burning the mattress, also burned part of the dwelling house of another, namely the floor. Therefore, (B) and (C) are incorrect.

In order to protect its citizens from "loan sharks" and unscrupulous business practices, a state had an anti-usury statute that prohibited loans for an interest rate in excess of 12%. When interest rates rose throughout the nation and the federal reserve began lending money to banks at 11.5%, banks in that state could not profitably make mortgage loans. Consequently, only extremely wealthy persons who could afford to pay cash for a home were able to purchase homes or farms in the state. If an appropriate party challenges the constitutionality of the anti-usury statute, how should the court rule? (A) The statute is constitutional, because the statute does not conflict with a specific federal statute. (B) The statute is constitutional, because the state acted properly under its police power in enacting the statute. (C) The statute is unconstitutional, because the statute denies nonwealthy people equal protection of the law. (D) The statute is unconstitutional, because it restrains interstate commerce.

(B) The court should rule that the statute is constitutional. A state has the right, under its police power, to enact legislation for the health, safety, and welfare of its citizens, provided that such legislation is not in areas reserved to the federal government or preempted by federal legislation, and provided that a state does not unduly restrain interstate commerce. The legislation in question affects only that state's banks and does not discriminate against other banks. The political process is considered the adequate method for bringing such statutes as the anti-usury statute in line with contemporary conditions. (A) is incorrect because even if a state statute does not actually conflict with a federal statute, the state statute may still fail under the Supremacy Clause if it prevents achievement of federal objectives or Congress has indicated an intent to preempt an entire field. Thus, (B) is a better answer than (A). (C) is incorrect because income level is not a "suspect" category, and the fact that the super-rich can purchase homes while everyone else cannot is an incidental result of economic conditions. Regardless of the interest rate, there will always be some MPQ 100 preview diagnostic exam and divider P.indd 88 12/22/2015 4:13:33 PM MBE PREVIEW DIAGNOSTIC EXAM 89. citizens who cannot obtain mortgages. (D) is incorrect because the legislation affects only that state's banks. There may be a slight incidental effect on interstate commerce, but the legislation in no way discriminates in favor of that state's banks.

A plaintiff filed a breach of contract action against a defendant in federal district court, invoking the court's diversity of citizenship jurisdiction. The defendant filed an answer denying the material allegations in the complaint. Approximately two months after the answer was served, the court entered a scheduling order that required the parties to complete all discovery within 10 months after the entry of the scheduling order. Two months later (four months after service of the answer and two months into the discovery period), the defendant sought to amend his answer to add an affirmative defense that the plaintiff's claim was barred by the statute of limitations. Which of the following statements is correct regarding the defendant's right to amend his answer to add an affirmative defense? (A) The defendant has a right to amend his answer any time before trial. (B) The defendant must obtain leave of the court to amend his answer, but the court should freely grant leave. (C) The defendant may amend his answer to add the affirmative defense only if he can show he could not, with due diligence, have discovered the defense prior to serving his answer. (D) The defendant may not amend his answer more than 21 days after serving it.

(B) The defendant may amend with leave of the court. A party may amend a responsive pleading of right within 21 days after serving it. Thereafter, according to Federal Rule 15, the party may amend only with consent of all parties or with leave of the court, but the "court should freely grant leave when justice so requires." With so much time left for discovery and before trial, a court would almost certainly grant leave to amend. (A) is incorrect because, as stated, the time for amendment as of right is 21 days after serving it, not any time before trial as (A) implies. (C) is incorrect in that due diligence in discovering the defense need not be shown. (D) is incorrect. The 21-day period applies to amendment as of right, but a court may grant leave to amend after that period.

A stockbroker visited a customer at the customer's office and sold her some securities. Coincidentally, three days after the sale, the stockbroker and customer were involved in a car accident on the freeway into the city. After the securities dramatically declined in value, the customer determined that the broker violated federal securities statutes when the broker sold the securities. The customer and the broker are citizens of the same state. The customer filed an action against the broker in federal district court asserting a claim for $70,000 for the broker's violation of federal securities statutes and a claim for $4,000 for the broker's negligence in damaging the customer's car. May a federal court hear these claims together? (A) No, because unrelated claims may not be joined in the same action. (B) No, because the federal court lacks subject matter jurisdiction over the negligence claim. (C) Yes, because the amount in controversy is irrelevant in federal question cases. (D) Yes, because the court has federal question jurisdiction over the statutory securities claim and supplemental jurisdiction over the negligence claim.

(B) The federal court may not hear these claims together because there is no supplemental jurisdiction over the negligence claim. Generally, every claim in federal court must have a basis for federal subject matter jurisdiction. There are two main bases for federal subject matter jurisdiction—diversity of citizenship jurisdiction and federal question jurisdiction. Once a claim is in federal court, supplemental jurisdiction sometimes may be used to have a claim heard. Diversity of citizenship jurisdiction is available when (i) there is complete diversity of citizenship, meaning that each plaintiff is a citizen of a different state from every defendant; and (ii) the amount in controversy exceeds $75,000. A natural person's citizenship is the state that is the person's domicile. In the instant case, the facts state that the broker and customer are from the same state. As a result, complete diversity does not exist. Furthermore, although the customer may aggregate all the claims he has against the broker, the aggregate amount ($74,000) does not meet the minimum amount in controversy requirement. For these reasons, subject matter based on diversity is not available. Federal question jurisdiction is available when the plaintiff, in his well-pleaded complaint, alleges a claim that arises under federal law. Anticipation of a federal defense or the fact that federal law is implicated by the plaintiff's claim do not give rise to federal question jurisdiction; the plaintiff's claim must arise under federal law. Here, the customer alleges that the broker violated federal securities law. That is sufficient to invoke federal question jurisdiction over the securities claim (which does not have an amount in controversy requirement or a complete diversity requirement). However, the claim for damages to the customer's car is a state law claim, and federal question jurisdiction is not available. Thus, to be heard, the negligence claim must invoke supplemental jurisdiction. When the federal court has subject matter jurisdiction over one claim, it has discretion to exercise supplemental jurisdiction over related claims that derive from the same common nucleus of fact and are such that a plaintiff would ordinarily be expected to try them in a single judicial proceeding. Here, though, the customer's negligence claim is not related in any way to the customer's claim for violating federal securities law. As a result, supplemental jurisdiction is not available. Thus, (B) is the correct choice and (D) is incorrect. (C) is incorrect even though it contains a true statement of law because there is no supplemental jurisdiction and thus no federal subject matter jurisdiction. (A) is too broad of a statement. When dealing with a single plaintiff against a single defendant, the plaintiff is allowed to join any number and type of claims against the defendant. Thus, if subject matter jurisdiction requirements could have been satisfied, the customer here could have joined all the claims he has against the broker. (When multiple plaintiffs or multiple defendants are involved, it is essential only that at least one of the claims arise out of a transaction in which all were involved.)

A gardener entered into a contract with a landowner under which, for a monthly fee of $200, the gardener would provide the landowner with monthly gardening services for the calendar year. On June 1, the gardener told the landowner that she could no longer afford to work for $200 a month and demanded that she be paid $400 a month. The landowner refused, stating that he already had a valid contract with her and that he did not have to pay any more. The gardener again insisted that she would not work for less than $400. As a result, the gardener and the landowner agreed in writing on July 1 that the gardener would thereafter be paid $400 a month for the gardening services and that the term of the agreement would be extended through the end of next year. Is the gardener legally entitled to receive a monthly fee of $400 from the landowner after July 1 of this year? (A) Yes, but only for the period from January 1 to December 31 of next year. (B) Yes, because the gardener and the landowner mutually agreed to modify their original contract. (C) No, because the gardener used economic duress in order to obtain the landowner's agreement to the modification. (D) No, because the gardener was under a preexisting duty to perform the gardening services for $200 a month.

(B) The gardener is entitled to receive the fee. The common law rule is that a promise to modify a contract must be supported by consideration. Here, the gardener agreed to work an extra year in return for the landowner's promise to pay extra money. In the absence of facts indicating duress or unconscionability (facts not present here), courts will not inquire into the adequacy of the consideration. There is no indication that the landowner had no "meaningful choice" but to accept the price increase, and because the landowner received a year's extension in exchange, there was adequate consideration to support a modification. (A) is incorrect because, as discussed above, the modification is valid and the price increase can take effect as agreed. (C) is incorrect for two reasons: First, economic duress generally is not a good defense to contract unless caused by the party seeking to enforce the contract. Second, as mentioned above, there are no facts indicating that the landowner had no "meaningful choice" but to accept the proposed modification; nothing indicates that the landowner could not have found another gardener or could not have sued the gardener for breach if she had not performed the original contract in the absence of modification. (D) is incorrect because the gardener did not merely promise to perform a preexisting duty in exchange for the promise to pay $200 a month more. She promised a year's extension of services, and this is additional and valuable consideration.

An employee of a construction company working in a high crime neighborhood negligently cut through power cables. The accident knocked out power to the area and disabled a home's security system. The homeowner, who was out of town, received an alert from the security company advising her to arrange for someone to go to the house and restart the system with a backup power source. The homeowner got distracted soon after getting the message and neglected to contact anyone. That evening, a burglar broke into the home while power was still off in the neighborhood. Because the security system was not working and the alarm did not go off, the burglar was able to steal jewelry and other valuables from the home and escape. The homeowner sued the construction company for the loss of her valuables in a jurisdiction that has adopted pure comparative negligence rules. Is the homeowner likely to recover? (A) Yes, because the homeowner's negligence contributed the least to her loss. (B) Yes, because the conduct of the construction company's employee created the opportunity for the burglar to steal the valuables. (C) No, because the homeowner's negligence was a superseding cause of her loss. (D) No, because the burglar committed a criminal act that was a superseding cause of the loss.

(B) The homeowner is likely to recover from the construction company. The general rule of proximate cause is that the defendant is liable for all harmful results that are the normal incidents of and within the increased risk caused by his acts. In indirect cause cases, an independent intervening force may be foreseeable where the defendant's negligence increased the risk that these forces would cause harm to the plaintiff. Even a criminal act by a third party will not cut off the defendant's liability if the defendant's negligence created a foreseeable risk that a third person would commit the crime. Here, the construction company negligently cut the power in a high crime neighborhood, increasing the risk of criminal conduct occurring. But for the power being cut, the homeowner's valuables would not have been stolen. A jury is likely to find that the burglar's intervening act was sufficiently foreseeable that the construction company will be held liable for at least some of the damages suffered by the homeowner. (A) is incorrect because, under pure comparative negligence rules, the homeowner could recover some of her damages even if her negligence was deemed to be greater than that of the defendant. (C) is incorrect because superseding cause analysis does not apply to the plaintiff's negligence. The homeowner's failure to have the security system reset is an issue of contributory negligence, which under the jurisdiction's pure comparative negligence rules is not a complete defense. (D) is incorrect because, as discussed above, the burglar's conduct probably would be deemed foreseeable and therefore not a superseding force that cuts off the construction company's liability

A hunter drove to one of his old spots for target shooting. When he arrived at the spot, he noticed that an adjacent area that had formerly been an open field now contained a new housing tract with a playground where children were playing. The hunter tacked a paper target to a tree that was located between the playground and the point from which he planned to shoot. He fired a number of shots at the target, and all but one hit the target. The one shot that missed that went wide of the target, ricocheted off a tree, and flew into the playground, striking a child in the head, killing the child. If the hunter is tried for the death of the child, what is the most serious crime of which he can be convicted? (A) Murder, because the hunter's intent to shoot the child can be inferred from his conduct. (B) Murder, because the hunter acted with wanton recklessness when he chose to shoot the gun in an area near the playground. (C) Manslaughter, because the hunter did not intend to kill the child. (D) Neither murder nor manslaughter, because the hunter's shooting was negligent, and mere negligence is insufficient to support conviction for a homicide crime.

(B) The hunter can be convicted of murder. Conviction for common law murder requires malice. At common law, malice is: (i) the intent to kill (express malice); (ii) intent to inflict great bodily injury; (iii) reckless indifference to an unjustifiably high risk to human life ("depraved heart"); or (iv) intent to commit a felony ("felony murder"). The hunter did not intend to kill a child when he fired at the target, so he did not meet the specific intent test. Thus, (A) is incorrect. Malice is also shown when a person acts with a wanton and reckless disregard for human life. Based on these facts, the hunter clearly acted with malice by firing a high-powered gun in front of an occupied playground; thus, (B) is the correct answer. (C) is incorrect, given that specific intent is not the only way to show malice. (D) is incorrect because the hunter's actions go far beyond simple negligence.

A mechanic noticed that his neighbor had a "for sale" sign on his old car. The mechanic thought it would be fun to have an old car to fix up and customize, and he asked his neighbor how much he wanted for the car. The neighbor told him that he would sell the car for $400, and the mechanic responded that he would buy the car for $400. They agreed that the mechanic would come to the neighbor's house by 6 p.m. the next day with the money. At 9:15 the next morning, the neighbor called the mechanic and told him that when he agreed to the sale the preceding day, he forgot that he had just put two new tires on the car and, therefore, would need to be paid an extra $50. The mechanic agreed to bring $450 in cash to his neighbor's house at six o'clock that night. Is the mechanic legally bound to pay his neighbor the additional $50? (A) Yes, because the original contract was not in writing. (B) Yes, because the contract, as modified, does not exceed the minimum dollar amount required to invoke the Statute of Frauds. (C) No, because no additional consideration was given for the oral modification. (D) No, because contracts for the sale of goods can only be modified by merchants.

(B) The mechanic is bound to pay the neighbor the additional $50. A contract for the sale of goods may be modified without consideration to support the modification if the modification was sought in good faith. No writing is required under the Statute of Frauds unless the contract, as modified, is within the Statute. Here, the parties formed a contract for the sale of goods (a car) when the mechanic agreed to buy the car for $400. (B) is correct because the contract as modified is under $500, so it is enforceable, even though it is not evidenced by a writing. (A) is incorrect because the fact that the original contract was not in writing is irrelevant. The original contract was for the sale of goods under $500 in value, and the Statute of Frauds does not apply. Thus, the mechanic could have enforced the original oral contract if he had not agreed to the modification. (C) is incorrect because under U.C.C. section 2-209, no consideration is needed for the modification of a contract for the sale of goods. (D) is incorrect because all Article 2 contracts can be modified, if done in good faith.

The plaintiff and defendant in a lawsuit were in an accident in which the plaintiff was injured. As a result of the accident, the plaintiff incurred medical expenses of $100,000. At the time of the accident, the plaintiff and defendant both lived in State A. Before the action was filed, the plaintiff moved permanently to State B. The plaintiff then filed a negligence action against the defendant in federal district court, with subject matter jurisdiction being based on diversity of citizenship. After the action was filed but before the defendant was served with process, the defendant was transferred by his employer and moved permanently to State B. For purposes of evaluating the court's diversity of citizenship jurisdiction, what are the citizenships of the two parties? (A) Both are citizens of State A. (B) The plaintiff is a citizen of State B and the defendant is a citizen of State A. (C) The plaintiff is a citizen of State A and the defendant is a citizen of State B. (D) Both are citizens of State B.

(B) The plaintiff is a citizen of State B and the defendant is a citizen of State A. In addition to an amount in controversy that exceeds $75,000, diversity of citizenship jurisdiction requires complete diversity, meaning that each plaintiff must be a citizen of a different state from every defendant. Whether complete diversity exists is determined when the suit is filed, not when the cause of action arose or when the defendant is served with process. The citizenship of a natural person is the state in which he is domiciled. A new state citizenship may be established by (i) physical presence in a new place; and (ii) the intention to remain there permanently. In this question, the plaintiff was originally from State A, but then moved permanently to State B before suit was filed. After suit was filed, the defendant also moved to State B from State A. Because the plaintiff's move to State B was before he filed suit, he is considered to be a citizen of State B for purposes of diversity jurisdiction, whereas the defendant is considered to be a citizen of State A because his move did not occur until after suit was filed. Thus, complete diversity exists. Choices (A), (C), and (D) are incorrect for the reasons stated above.

A homeowner leased his home to a tenant for three years. The following year, the homeowner conveyed the house to a buyer, who never recorded her deed nor did anything with regard to the house. The tenant continued paying rent to the homeowner. Three months after the conveyance to the buyer, the homeowner conveyed the property to his proctologist, who knew nothing of the prior conveyance to the buyer. The homeowner took the proctologist's money and skipped town. The proctologist told the tenant that he now owned the house and that all rents should be paid to him. The tenant complied. Six months later, the proctologist went to his local bank for a loan. He offered to put up the property as security. The bank discovered that the proctologist had never recorded his deed and that, just two weeks prior to his loan application, the buyer had recorded a deed to the house that bore an earlier date than the deed the proctologist had shown the bank. Because of this cloud on the title, the bank refused the loan request. When the tenant discovered this, she quit paying rent to the proctologist. The state has a recording statute that provides, "a conveyance of an interest in land, other than a lease for less than one year, shall not be valid against any subsequent purchaser for value, without notice thereof, unless the conveyance is recorded." If the proctologist sues the tenant to compel the payment of rent, is the proctologist likely to win? (A) Yes, because the tenant is estopped from denying a landlord-tenant relationship with the proctologist, since she had paid rent for many months. (B) Yes, because the proctologist was a bona fide purchaser when he bought the property from the homeowner. (C) No, because the proctologist failed to record his deed to the property. (D) No, because the proctologist did not have good title to the property and cannot demand rent from tenants in possession

(B) The proctologist will likely win. When the proctologist bought the property from the homeowner, he was a bona fide purchaser who gave value and who had no notice of the earlier sale to the buyer. Not only did the proctologist not have actual notice of the earlier sale, he did not have constructive notice either because the buyer did not record before the proctologist bought. The recording statute in the jurisdiction is a notice statute. In a jurisdiction with a notice recording statute, a subsequent purchaser who gives value and takes without notice wins over the earlier grantee. If the facts had shown a jurisdiction with a race-notice recording act, the proctologist would have been in trouble. With race-notice, the proctologist would not only have to take without notice, he would have to be the first to record. Because the facts do not show that the proctologist recorded at all, he would lose. But because this is a notice act jurisdiction, the fact that the buyer finally recorded before the proctologist is irrelevant. (A) is incorrect. Simply because a tenant pays rent to someone who the tenant thinks owns the property does not create an estoppel requiring the MPQ 100 preview diagnostic exam and divider P.indd 81 12/22/2015 4:13:33 PM 82. MBE PREVIEW DIAGNOSTIC EXAM tenant to continue paying rent to that person. The proctologist will win, but not for this reason. (C) is incorrect because it is not necessary for a grantee to record a deed to give the grantee legal title to the property. Recordation is necessary only to protect against subsequent purchasers from the same grantor. Unrecorded deeds are perfectly valid as long as the recording act does not dictate otherwise. (D) is incorrect. The proctologist did in fact have good title to the property because, as a bona fide purchaser, he was protected by the notice recording act.

The defendant was charged with forging the signature of her employer on several checks made payable to her best friend, a co-worker. After discovering that the checks had been forged, the employer called the defendant into his office and told her that he regretted trusting her with the task of paying his bills and could not believe that his two most trusted employees devised a scheme to defraud him. The defendant did not respond to the statement and walked out of the room. At trial, the prosecution wants to offer the defendant's silence as evidence of her guilt. How should the court rule? (A) Admissible under an exception to the hearsay rule for implied statements against interest. (B) Admissible nonhearsay. (C) Inadmissible, because failure to reply to an accusation in a criminal case cannot be used as an implied admission of a criminal act. (D) Inadmissible as violating the defendant's right against self-incrimination.

(B) The silence is admissible nonhearsay as a statement by a party-opponent (commonly called an admission). Under the Federal Rules of Evidence, silence in response to an accusatory statement can be considered an implied admission if the party heard and understood the statement, was capable of denying the statement, and a reasonable person would have denied the accusation under the same circumstances. Here, there is nothing to indicate that the defendant did not hear or understand the statement or that she was incapable of denying it, and a reasonable person who was innocent would have in fact denied such an accusation. Thus, the defendant's silence is admissible as an admission, which is nonhearsay under the Federal Rules. (A) is incorrect because an implied admission by silence is not considered hearsay at all, and therefore is not an exception to the hearsay rule. (C) is incorrect because the rule against admissibility applies only if the accusation or statement is made by the police; here, the accusation was made by the employer. (D) is incorrect because the Fifth Amendment right against self-incrimination, which provides that a defendant cannot be compelled to testify against herself, applies only after the initiation of a criminal proceeding. Here, the statement was made prior to the institution of criminal proceedings against the defendant.

An owner of 40 acres of mountain land sold the western 20 acres to a buyer. Because no access to any public road existed on the western side of the property, the deed conveying title to the buyer included an easement for ingress and egress that ran along the southern border of the owner's land. This deed was duly recorded. About the same time that this sale took place, the county extended the public road so that it abutted on the buyer's 20 acres. The buyer then built a cabin on the property. Later, the owner and the buyer both sold their 20 acres to the owner's cousin. The cousin then sold the 20 acres acquired from the buyer to his lawyer and sold the other 20 acres to a doctor. All deeds involved in the various conveyances of the 20-acre parcels were validly recorded. Neither the cousin-lawyer deed nor the cousindoctor deed made any mention of easements or rights-of-way. In fact, no use was ever made of the easement. A few years later, the lawyer sold his 20 acres to a development company that wished to build a hunting lodge on the property. The development company now wants to construct a road across the doctor's property in the manner contemplated in the deed from the original owner to the buyer. What is the strongest argument as to why the development company should not be permitted to construct the road? (A) Any easement that might have existed was extinguished by abandonment because of nonuse. (B) Any easement that once existed was terminated by merger. (C) The county's subsequent expansion of the public road removed the necessity of using the easement across the eastern parcel. (D) Neither the cousin-doctor deed nor the cousin-lawyer deed made any mention of the easement.

(B) The strongest argument is based on merger. When the cousin bought both parcels from the owner and the buyer, the dominant (buyer) and servient (owner) estates came together in the same owner and the easement was terminated by merger. Once terminated by merger, the easement does not revive when the ownership is later split. (A) is incorrect. An easement may be terminated by abandonment, but mere nonuse is not enough. The holder of the dominant estate must physically manifest an intent to abandon. The facts do not show that any holder of the western parcel did anything to indicate this intent. (C) is incorrect. The easement granted in the deed from the owner to the buyer was a valid express easement. No implied easement arose because there was no necessity for one. The later construction of the public road can have no effect if there had been no valid implied easement in the first place. (D) is incorrect. The fact that the easement was not included in the subsequent deeds is irrelevant because the easement was in the owner-buyer deed, which was recorded properly in the chain of title. That puts all subsequent purchasers on notice of the easement, and it need not be put in subsequent deeds to be given effect. Exam Tip: Multiple transfers can be confusing on the exam. A technique to aid analysis is to make a quick chart in the margin of the exam booklet. This would have clearly shown the merger and unity of ownership: West East Owner Owner Buyer Owner Cousin Cousin Lawyer Doctor Dev. Co. Doctor

A boater and a water skier were involved in a boating accident. Shortly after the accident, the water skier prepared a written summary of the events surrounding the accident. At trial two years later, the water skier is on the stand and is unable to accurately recall the details of the accident, even after reviewing the aforementioned account of the accident. Assuming a proper foundation is laid, may the summary of the accident be read into evidence? (A) No, because the best evidence is the writing itself. (B) Yes, because the water skier's memory of the actual event is insufficient. (C) Yes, because it refreshes the water skier's recollection. (D) Yes, even though hearsay, because the out-of-court declarant is on the stand and is capable of being cross-examined.

(B) The summary of the accident may be read into evidence. If the witness on the stand has insufficient recollection to testify to a relevant event fully and accurately, Federal Rule 803(5) permits the introduction of an out-of-court written record of the event made by the witness at a time when the witness's memory of the event was fresh. The fact that the witness's memory of the actual event is insufficient even after reviewing the summary would satisfy that standard, assuming a proper foundation has been laid for reading the statement into evidence. If the past recollection recorded is admissible, the record itself may not be admitted; unless it is offered into evidence by the adverse party, the offering party may only read the record to the jury, as here. (A) is incorrect. There is no question that the document which is read must satisfy the best evidence rule and be either the original or a duplicate, or there must be a satisfactory excuse for nonproduction of an original. There is no indication in this case that the summary in the witness's possession would fail to satisfy the best evidence rule. It appears to be her original notes. The real question here is whether the document in question is admissible despite the hearsay rule. (C) is incorrect. If a written record is used to refresh the memory of a witness who then proceeds to testify from her own memory as to the matter, then the memorandum is not evidence coming within the past recollection recorded exception under Federal Rule 803(5). A document which is used to refresh a witness's memory may not be read to the jury. Instead, the witness will testify from her refreshed memory. (D) is incorrect. The hearsay rule applies in general to any out-of-court statement, whether or not the declarant is available for cross-examination. However, this statement comes within the hearsay exception for a past recollection recorded under Federal Rule 803(5). Under that Rule, the contents of the memorandum may be read to the jury. Exam Tip: The difference between present recollection refreshed and past recollection recorded is a recurring exam favorite.

The police suspected that an 18-year-old living with his mother was selling marijuana to his high school classmates. The police went to the house without a search warrant and secured the mother's permission to search the teen's bedroom. In the bedroom, they found a locked suitcase under the teen's bed. The mother said that it was her son's suitcase but that they could search it. The police then broke the lock, opened it and found a substantial amount of marijuana. The police charged the teen with possession of marijuana with intent to distribute it. If the teen should bring a pretrial motion to suppress the marijuana, how should the court rule on the motion? (A) Grant it, because the mother was not informed of her right to refuse consent to the search. (B) Grant it, because the mother's consent to search the suitcase was invalid. (C) Deny it, because the mother had authority to consent to a search of anything within her home. (D) Deny it, under the doctrine of in loco parentis.

(B) The teen's motion should be granted. A search is valid if the police reasonably believe that they obtained valid consent for the search. Generally, a homeowner has authority to consent to a search of her own home, as long as she has apparent access to the place searched. Here, nothing indicates that the teen's bedroom door was locked. Thus, the mother had apparent authority to search the teen's room. However, it appears unreasonable for the police to believe that the mother had authority to consent to a search of the suitcase because it was locked, and the police had to break the lock to open it (indicating that the mother did not have a key for the lock and so likely did not have access to the contents of the suitcase). The parent-child relationship does nothing to change this analysis. A parent of an adult teenager does not have automatic authority to search the adult teen's locked things (the rule probably is different for young children). (A) is incorrect. To be effective, a consent to search must be voluntary and given under no threat or compulsion. However, the police are under no obligation to inform the defendant or other person of the right to refuse entry. Therefore, this search is not invalid because the mother was not informed of her right to refuse entry. The issue is whether her consent was sufficient to allow the police to search the son's suitcase. (C) is incorrect because, as explained above, a homeowner does not necessarily have authority to consent to a search of everything in her home. Where it is apparent that she does not have access (e.g., when something is locked and she does not have a key), the police may not reasonably believe she may consent to the search of the locked area. (D) is incorrect. The doctrine of in loco parentis is a tort doctrine under which the state is charged with a parent's responsibilities over a child in its care. The doctrine does not apply here.

The defendant was on trial for shooting and killing a man at a restaurant after a violent argument. The prosecution calls a witness to testify that she was standing in the parking lot, and, as the defendant was leaving the restaurant, he dropped a gun into a nearby garbage can. Is the witness's testimony regarding the defendant's conduct admissible? (A) Yes, as hearsay falling within the state-ofmind exception. (B) Yes, because nonassertive conduct is not hearsay, and it is otherwise admissible. (C) No, because nothing was said. (D) No, because the defendant's acts constitute assertive conduct.

(B) The witness's testimony regarding the defendant's conduct would be admissible. The defendant's acts constitute nonassertive conduct. Alternatively, this could be considered an admission by conduct. Therefore, the testimony is direct, relevant, nonhearsay evidence. Because it is not hearsay, (A) is incorrect. (C) is incorrect because evidence is not limited to testimony regarding spoken words; a witness may testify as to what she saw. (D) is incorrect because the defendant did not intend his conduct to be a substitute for words.

A mountain lodge catered to cross-country skiing enthusiasts. Every winter, the owner of the lodge, along with his assistants, marked and groomed trails on his extensive lands. He charged a fee for a day's use of the trails. He also supplied equipment for those who needed it and provided lodging and meals for skiers staying more than one day. A woman on her first cross-country skiing outing was striding along a groomed trail when, with a loud crack, the rotten limb of a huge oak under which the trail passed separated from the tree because of the load of ice and snow it bore. It came crashing down on top of the woman, killing her instantly. The woman's heirs brought a wrongful death action against the lodge owner in a jurisdiction applying traditional rules for landowners and possessors of land. If the lodge owner prevails, what is the most likely explanation? (A) The oak limb was a natural condition of the land. (B) The lodge owner did not know that the oak limb was rotten and was likely to fall. (C) The lodge owner could not reasonably have discovered the rotten limb by inspecting his property. (D) The ice and snow was an intervening force that caused the limb to fall.

(C) If the lodge owner prevails, it will be because he made a reasonable inspection without discovering the danger. In a jurisdiction applying traditional rules for landowners and possessors of land, a landowner owes a duty to invitees to warn of or otherwise make safe dangerous natural conditions of which he knows or could reasonably discover by inspection. (A) is wrong because the fact that the tree limb was a natural condition does not relieve the lodge owner of this duty. (B) is wrong because actual knowledge is not necessary; it is sufficient to impose liability if the dangerous natural condition would have been discovered by a reasonable inspection. (D) is wrong because the fact that other factors contributed to the danger does not negate the landowner's liability because those factors were foreseeable.

The police set up an undercover "sting" operation in which they posed as fences of stolen property and bought and sold such property to anyone who came into their downtown warehouse. A defendant is being prosecuted for receiving stolen property in connection with his arrest by the undercover operatives, and the prosecution attempts to introduce a videotape showing the defendant offering to sell a television set to one of the police officers. If this evidence is held to be inadmissible, what is the most likely reason? (A) It is hearsay not within any exception. (B) It violates the defendant's privilege against self-incrimination. (C) A proper foundation was not established for its introduction into evidence. (D) Criminality may not be proven by specific instances of misconduct.

(C) Lack of a proper foundation is the most likely reason for this evidence to be held inadmissible. No single facet of the law of evidence provides a ready answer to this question. Thus, it is useful to analyze each choice and arrive at the correct answer by a process of elimination. (A) is incorrect because the videotape showing the defendant offering a television set to one of the police officers is not hearsay evidence. Even if the defendant's conduct on the videotape is considered assertive conduct and is thus a "statement" within the meaning of the hearsay rule (much like nodding one's head to indicate "yes" is a statement), under the Federal Rules a statement of a party-opponent is admissible nonhearsay. [Fed. R. Evid. 801(d)(2)] Because the defendant is a party to this criminal prosecution, his conduct, if considered a statement, would be admissible. It is not clear, however, that the defendant's conduct on the videotape is assertive conduct. If it MPQ 100 preview diagnostic exam and divider P.indd 77 12/22/2015 4:13:32 PM 78. MBE PREVIEW DIAGNOSTIC EXAM is considered nonassertive conduct—conduct that is not intended to communicate a thought—it would not be a statement within the meaning of the hearsay rule. [Fed. R. Evid. 80l(a)] Thus, this is an alternative reason, albeit a less clear one, why (A) is incorrect. (B) is incorrect because the privilege against self-incrimination applies to compulsory self-incrimination. The defendant was not forced to incriminate himself. He voluntarily took an action (offering to sell a television set to one of the police officers) that helps establish that he is guilty of receiving stolen property. Such an act is not covered by the self-incrimination privilege. (D) is also incorrect because specific instances of misconduct pertaining to an event (here, the receipt of stolen property) are frequently used to prove criminality. If (D) said that specific instances of past misconduct pertaining to a defendant's character (i.e., propensity evidence) were inadmissible, that would be true as an abstract statement of law, but (D) does not say that, nor do the facts in the question remotely relate to propensity evidence. Thus, (A), (B), and (D) are clearly incorrect. (C), on the other hand, is a plausible answer. If the videotape is held to be inadmissible, one reason could be that a proper foundation had not been established for its introduction into evidence. A videotape, like other forms of tangible evidence, must be authenticated before it can be admitted. The failure to properly authenticate the videotape is one reason why it could have been held inadmissible.

A pedestrian and driver were involved in an automobile crash in the Eastern District of State A. The pedestrian resides in the Southern District of State A, and the driver resides in the District of State B. The pedestrian filed a negligence action in the United States District Court for the Southern District of State A properly based on diversity jurisdiction. The driver responded by filing a motion to dismiss the action for improper venue or, in the alternative, to transfer the action to the Northern District of State A. What options for disposition are available to the court? (A) Deny dismissal or transfer because the Southern District of State A is a proper venue. (B) Either dismiss the action or grant transfer to the Northern District of State A. (C) Deny transfer of the action to the Northern District of State A because the Northern District of State A is not a proper venue. (D) Grant transfer of the action to the Northern District of State A only if it determines that the Northern District is a fair and convenient forum.

(C) The court may not transfer the action. Federal venue in diversity actions is proper in (i) the district in which any defendant resides if all defendants reside in the same state; and (ii) the district in which a substantial part of the events or omissions giving rise to the claim occurred. The Southern District of State A is not a proper venue because it is neither the defendant's residence nor a place where substantial events occurred giving rise to the action. As a result, the court must either dismiss, or in the interests of justice, transfer the case to a district in which the action could have been brought on the defendant's timely objection to venue. Here, however, the court may not transfer to the Northern District of State A because that district also is improper, as it is also neither the defendant's residence nor a place where substantial events occurred giving rise to the action, and thus the action could not have been brought there. Thus, (B) is incorrect. (A) is incorrect because, in federal court, venue may not be based on the plaintiff's residence. (D) is incorrect because the venue statute does not allow the judge to make such a discretionary call; rather, the district to which venue is transferred ordinarily must be a place of proper venue.

A large apple farm sells its product to various producers in the area who market apple products for commercial use. Prior to the harvesting season—the period when apple producers normally negotiate with processors and wholesalers for sale of the apples to be harvested—the farm was contacted by a health foods corporation that agreed in writing to purchase all of the farm's harvested apples. After the contract was signed, but before the harvest had begun, a rival apple farm offered to sell its apples to the corporation for 20% less than the price agreed to by the first farm. The corporation signed a contract with the rival farm for the purchase of raw apple products early enough before the beginning of the harvest so that the first farm had plenty of time to obtain another buyer for its apples. The corporation informed the first farm that its requirements for apples for the next year would be about 5% of the farm's total production. If the first farm seeks specific performance of its contract with the corporation, which party is likely to prevail? (A) The farm, because performance of the corporation-rival farm contract will automatically breach the first farm's contract. (B) The farm, because its contract was prior in time to the rival farm's contract. (C) The corporation, because the first farm can resell its apples elsewhere and then seek damages, if any, from the corporation. (D) The corporation, because a seller of goods cannot obtain specific performance.

(C) The court should find for the corporation. Under the U.C.C., a seller may obtain specific performance (i.e., force the goods on the buyer) where the buyer has not yet accepted the goods only if the seller is unable to resell the goods at a reasonable price or the goods have been lost or damaged after risk of loss has passed to the buyer. Because the first farm has plenty of time to find another buyer and its apples are of a high quality, it will be able to resell the apples and obtain damages, but it cannot obtain specific performance. (A) is incorrect because mere breach of contract is not a sufficient basis for specific performance. (B) is evidence of a breach, namely that the corporation had an existing contract with the first farm at the time it signed the contract with the rival farm, but it does not address the issue of specific performance. (D) is an incorrect statement of the law. Specific performance is an equitable remedy that is available when the legal remedy is inadequate. However, because the first farm can easily resell its products, monetary damages are sufficient.

The plaintiff in a negligence action in federal district court was injured when the defendant's car crashed into his. The defendant's friend was a passenger in the defendant's vehicle at the time of the accident. The defendant's attorney sent an email to the passenger asking the passenger to describe in detail what the passenger remembered about the accident and the events leading up to it. In response, the passenger sent the defendant's attorney an email describing the events. The plaintiff served on the defendant the following request for documents: "Please produce for inspection or copying any and all statements obtained by you or your attorney that relate in any way to the events and/or issues that are the subject of this legal action." Must the defendant produce the passenger's statement? (A) Yes, because the statement is relevant to the parties' claims and defenses. (B) Yes, because the work product doctrine does not apply to emails and other digitally stored information. (C) No, because the statement is protected from discovery pursuant to the work product doctrine. (D) No, because the statement is privileged.

(C) The defendant does not need to produce the passenger's statement. Work product prepared in anticipation of litigation is discoverable only on a showing of substantial need and undue hardship in obtaining the substantial equivalent of the work product. Here, there is no indication that the passenger is unavailable or cannot recall the accident. Thus, it is unlikely that the plaintiff will be able to show substantial need and undue hardship in obtaining a statement from the passenger as to her recollection of the accident. As a result, the statement is protected from discovery. (Note that this analysis is applicable to the statement only; the existence of the passenger as a witness must be disclosed either as an initial disclosure—assuming the defendant is going to use passenger to support her claim or defense—or in response to a properly submitted interrogatory.) (A) is an overbroad statement of the requirements for discovery. Federal Rule 26(b)(3) specifically exempts documents prepared in the anticipation of litigation from discovery, and the passenger's statement falls into this category. (B) is an incorrect statement of the law; the fact that the statement was electronic does not prevent it from becoming protected under the work product doctrine. (D) is incorrect because the statement is not "privileged" per se (such as the doctor-patient evidentiary privilege), but rather is exempt from discovery under the work product doctrine under Federal Rule 26(b)(3). As described above, there are exceptions to the work product doctrine.

A bolt of lightning struck a tree, causing it to fall on a farmer's fence which enclosed a pasture containing the farmer's large bull. The bull escaped through the broken fence and entered the neighbor's property. It gored a hiker who was crossing the neighbor's property without permission. In the hiker's action against the farmer based on strict liability, is the hiker likely to prevail? (A) Yes, because the bull caused harm while trespassing on another's property. (B) Yes, because bulls have known dangerous propensities. (C) No, because a bull is a domestic animal. (D) No, because the hiker was a trespasser.

(C) The hiker will not prevail because strict liability does not apply to a bull, which is a domestic animal. The owner of a domestic animal, including a farm animal, is not strictly liable for injuries it causes, as long as the owner has no knowledge that the animal has abnormally dangerous propensities (i.e., propensities more dangerous than normal for that species). A bull is a domestic animal, and nothing in the facts suggests that the bull was more dangerous than normal for that type of animal. Hence, strict liability will not apply. (A) is incorrect because the rule for trespassing animals does not apply. The owner of a trespassing animal is strictly liable for harm done by the trespass as long as it was reasonably foreseeable. Here, the bolt of lightning caused the fence to break and allowed the bull to escape. This unforeseeable intervening force was the cause of the trespass; hence, the strict liability rule for trespassing animals does not apply here. (B) is incorrect because, as discussed above, strict liability does not apply for domestic animals with normal dangerous propensities. Only domestic animals with propensities more dangerous than normal for the species may subject the owner to strict liability. (D) is incorrect because the hiker's status as a trespasser on the neighbor's land is irrelevant as to the farmer's liability. If the hiker were a trespasser on the farmer's land, strict liability would not apply even if the bull were abnormally dangerous, but the farmer's liability is not affected by the hiker's status as to the neighbor. Note that if strict liability applied for harm from an animal trespassing on a neighbor's property, the hiker's status as a trespasser might be relevant because strict liability applies only to injured persons who were rightfully on the property. However, as discussed above in (A), that liability is inapplicable here because the bull's trespass was unforeseeable

A hotelier opening a new inn in the Pacific Northwest sent letters to all known hotel and motel suppliers on June 1, alerting them to his need for such items as ice buckets, televisions, linen, and mattresses. The hotelier received a letter dated June 8 from a hotel supply company, stating that the company had 250 ice buckets left in stock and will sell them to the hotelier for $1 each. The company added that it must receive the hotelier's answer by November 1 and will hold the ice buckets for the hotelier until then. On July 1, the company sold 200 of the ice buckets to a competing hotel chain, which had recently opened a hotel on the East Coast. On July 2, the company sent the hotelier a fax stating it had only 50 ice buckets left for sale. The hotelier received the fax that day, but put it aside and never read it. On July 10, the hotelier notified the company that he was accepting the company's offer to sell 250 ice buckets. The company, upon receiving the hotelier's acceptance, shipped the remaining ice buckets. The hotelier sues the company for failing to deliver all 250 ice buckets. Will the hotelier prevail? (A) No, because the hotelier is not a hotel supply merchant. (B) No, because the company's offer was to remain open for more than three months. (C) Yes, because the company promised in a signed writing to hold the offer open. (D) Yes, because the hotelier never read the company's July 2 fax.

(C) The hotelier will prevail. Ice buckets are movable goods; therefore, Article 2 of the U.C.C. applies. The June 8 letter from the supply company is a firm offer under U.C.C. section 2-205. No consideration is required, because the company is a "merchant" (i.e., one who ordinarily deals in goods of the kind sold) of ice buckets. Where a time period for the offer is stated, the MPQ 100 preview diagnostic exam and divider P.indd 69 12/22/2015 4:13:32 PM 70. MBE PREVIEW DIAGNOSTIC EXAM period of irrevocability is that period, except that the period cannot exceed three months. Here, the three-month period would end on September 8. The company's fax stating that it had only 50 ice buckets left to sell constitutes an invalid attempt at revocation, because it is within the threemonth period of irrevocability. (A) is incorrect because section 2-205 does not require that the offeree of a firm offer be a merchant; it requires that the offeror be a merchant, and the company is (see above). (B) is incorrect because a firm offer that states a period longer than three months is still firm for the first three months. (D) is incorrect because the hotelier's knowledge, or lack thereof, of the "revocation" of the company's offer is irrelevant because it was invalid; the fact that the company made a firm offer prevents it from revoking the offer within the stated time, not to exceed three months.

During a defendant's trial for murder, the state introduced circumstantial evidence tending to establish her guilt, but could not provide direct evidence that she had committed the murder. No one had seen the killer going into or coming out of the victim's office around 2:30 p.m., the time of the victim's death by strangulation. During her defense case, the defendant took the stand and testified that she could not have committed the charged murder, because on the day of the victim's death she had been on a business trip 3,000 miles away. The prosecution then called a rebuttal witness. The witness will testify that, at 1 p.m. on the day of the charged murder, he had put his hand on the defendant's knee while both sat at a bar across town. The defendant had then coquettishly removed his necktie and attempted to strangle him with it until bystanders pulled her away and she left the bar. The defendant objects to admission of the witness's testimony. How should the trial judge rule? (A) For the defendant, because she did not place her character at issue in the trial. (B) For the defendant, because the testimony is character evidence not directed at her character for truth and veracity. (C) For the state, because the testimony established that the defendant had the opportunity to commit the charged crime. (D) For the state, because the testimony established that the defendant has a character for violence.

(C) The judge should rule for the state. This question is not about propensity evidence. Rather, it is about evidence of prior bad acts to help prove something other than a person's propensity to act in a manner consistent with a character trait. Under Federal Rule 404(b), it is permissible to use evidence of a criminal defendant's prior bad acts to establish a motive for committing the crime, to help to identify him as the perpetrator of a crime, or, as here, to show opportunity to commit the crime. The defendant claims that she could not have committed the murder because she was on a business trip the day it occurred. The witness's testimony, however, pertaining to his encounter with the defendant in a crosstown bar on the day of the murder, is clearly relevant to show that the defendant had the opportunity to commit the murder. For this reason, (C) is correct. All the other answers are wrong because they mistakenly assume that the witness's testimony involves propensity evidence rather than prior bad acts evidence. (A) correctly states that the MPQ 100 preview diagnostic exam and divider P.indd 87 12/22/2015 4:13:33 PM 88. MBE PREVIEW DIAGNOSTIC EXAM defendant did not place her character at issue by testifying that she was across the country when the murder occurred. That would be a reason for excluding the witness's testimony if his testimony amounted to an attack on the defendant's character (i.e., propensity evidence). However, his testimony is to show opportunity; thus, it may be admitted. (B) correctly implies that propensity evidence can be used to impeach the testimony of a witness only if it addresses the witness's character for truth and veracity. The defendant was a witness in the trial. If the purpose of the witness's testimony had been to impeach her testimony by attacking her character for something other than truth or veracity, the testimony would be inadmissible. Because this evidence was admitted not to impeach but to show opportunity, (B) is incorrect. (D) correctly implies that the prosecution cannot introduce propensity evidence establishing a defendant's bad character until the defendant has placed her character in issue. The defendant's testimony did not place her character in issue. Thus, if the witness's testimony amounted to an attack on the defendant's character, the testimony would be inadmissible. Because the testimony was not an attack on the defendant's character, however, (D) is incorrect.

A tourist was struck and killed at a street crossing by an automobile. The accident was witnessed by a bystander and by the tourist's wife. The wife brought an action against the automobile driver for wrongful death, alleging excessive speed, failure to observe traffic signals, and defective brakes. A witness for the driver was called to testify that the driver enjoyed a reputation for being a safe and prudent driver based on his personal knowledge of the driver's driving habits. How should the trial judge rule on the witness's testimony? (A) Admissible, because, where there are no unbiased eyewitnesses to an accident, reputation as a safe driver may be used to prove the driver acted in conformity with that reputation at the time in question. (B) Admissible, because the witness first testified that he has personal knowledge of the driver's driving habits. (C) Inadmissible, because evidence of a reputation as a safe and prudent driver cannot be used to prove that the driver acted in conformity with that reputation at the time in question. (D) Inadmissible, because, in a civil case, character evidence may be used only after the plaintiff has attacked the character of the defendant.

(C) The judge should rule the witness's testimony inadmissible. This question involves the admissibility of propensity evidence. The driver's witness is prepared to testify that the driver has a reputation for being a safe and prudent driver, which the driver plans to use to prove that, because he has a propensity for driving safely, he was in fact driving safely at the time of the accident. This is a classic case of the impermissible use of propensity evidence. The general rule is that evidence of character traits (here, safety and prudence) is inadmissible in a civil case to prove that a party acted in conformity with those traits on a particular occasion. [Fed. R. Evid. 404(a)] This case fits squarely within that general rule. Thus, (C) is correct. All the other answers are wrong because they each assume that propensity evidence is admissible under some circumstances in a wrongful death action. (A) indicates that such evidence is admissible if there are no unbiased eyewitnesses to the accident. (B) indicates that such evidence is admissible because the witness has personal knowledge of the character traits (the driver's propensity for safety and prudence) that are the subject of his testimony. (D) indicates that such evidence is admissible if the plaintiff has attacked the character of the defendant. All of these answers miss the basic point—that propensity evidence is inadmissible in civil cases. Furthermore, (A) is wrong because it presumes that there are no unbiased witnesses to the accident, but the question makes clear that there was an unbiased witness: the bystander. (B) is also wrong because it indicates that the witness must first testify that he has personal knowledge of the driver's driving habits before testifying about the driver's reputation for safety and prudence. This answer confuses reputation testimony with opinion testimony. The witness's testimony pertained to the driver's reputation. To be a competent witness, the witness would have to have personal knowledge of the driver's reputation. He would not have to have personal knowledge of the driver's actual driving habits. Although perhaps not as nonsensical as (A) and (B), (D) is just as inaccurate, as it bears no similarity to anything in the rules of evidence pertaining to propensity evidence.

A corporation filed a breach of contract action against an individual in federal district court. Attached to the complaint was a written contract providing that the individual agreed to pay the corporation $100,000 over a period of years for specified services. The contract also contained the individual's signature. The individual told his lawyer that the signature was a forgery and that she had never signed or entered into the contract. On the basis of the individual's statement, the lawyer drafted, signed, and filed an answer. The answer denied the claim on a number of grounds and denied that the signature on the contract was that of the individual. The individual's lawyer later served on the corporation a request for production of documents. When the corporation objected to some of the requests, the lawyer filed a motion to compel production. Shortly before the hearing on the motion to compel, the individual advised the lawyer that the signature on the contract was in fact hers, but she and the lawyer agreed that she nonetheless should not be liable on the contract for other reasons. The lawyer thus continued to assert the motion to compel production. At the hearing on the motion, the lawyer referred to the complaint and the answer to justify the relevancy of the requests for production, but he did not mention the signature in any way. Has the lawyer violated Rule 11 of the Federal Rules of Civil Procedure? (A) No, because when the lawyer signed the complaint, he had evidentiary support in the expected testimony of the individual who denied signing the contract. (B) No, because the lawyer failed to state in the motion hearing that the signature on the contract was not that of the individual. (C) Yes, because when the lawyer referred to the answer in the motion hearing, he renewed his certification that facts in the answer had evidentiary support at the time of the hearing. (D) Yes, because the lawyer should not have signed the answer without having the individual sign a sworn statement that the signature on the contract was not hers.

(C) The lawyer has violated Rule 11. By "presenting" a document to the court, Rule 11 provides that a lawyer certifies that he believes the denials of factual contentions in the document are warranted on the evidence and that his belief is formed after a reasonable inquiry. One "presents" a document not only by signing or filing it, but also by "later advocating" it. Thus, Rule 11 imposes a continuing certification requirement, applicable any time a matter is presented to the court. At the time he signed the complaint, the lawyer believed there was evidence that the individual did not sign the contract based on the individual's own statement. When the lawyer referred to the answer in the motion hearing, the lawyer "presented" the answer to the court anew and renewed his certification, despite the fact that he never discussed the signature. At that time, the lawyer no longer believed that all the facts in the answer had evidentiary support. He knew that he lacked evidence to support the answer's denial that the signature on the contract was that of the individual. Thus, (C) is correct and (B) is incorrect. (A) is incorrect because it does not take into consideration the "later advocated" basis for presenting a document under Rule 11. (D) is incorrect because an attorney need not have his client sign a sworn statement for every fact that the client tells the attorney.

A developer prepared and recorded a subdivision plan, calling for 100 home sites on half-acre lots. There were five different approved plans from which a purchaser could choose the design of the home to be built on his lot. Each deed, which referred to the recorded plan, stated that "no residence shall be erected on any lot that has not been approved by the homeowners' association." A lawyer purchased a lot and built a home based on one of the approved designs. However, many of the lots were purchased by investors who wanted to hold the lots for investment purposes. Two years after the lots went on the market, one such investor sold her lot to an architect by a deed that did not contain any reference to the recorded plan nor the obligation regarding approval by the homeowners' association. In fact, because very few residences had been built in the subdivision since the lots were first available for purchase, no homeowners' association meetings had been held in two years. The architect began building a very modernistic house on her one-half acre. When the lawyer noticed the house being built, he brought an action to enjoin the construction. For which party will the court rule? (A) The architect, because her deed contained no restrictive covenants. (B) The architect, because any restrictive covenant in her deed can only be enforced by the opposite party to the covenant or that person's successor in title. (C) The lawyer, because the recorded subdivision plan, taken with the fact that all lots were similarly restricted and the architect had notice of this, gave him the right to enforce the covenant on her property. (D) The lawyer, because his deed contained the restrictive covenant.

(C) The lawyer will likely prevail. When a subdivision is created with similar covenants in all deeds, there is a mutual right of endorsement (each lot owner can enforce against every other lot owner) if two things are satisfied: (i) a common scheme for development existed at the time that sales of parcels in the subdivision began; and (ii) there was notice of the existence of the covenant to the party sued. Here, there was a common scheme evidenced by the recorded plan, and the fact that the covenant was in the architect's chain of title gave her constructive notice of the restriction. Therefore, not only does the covenant apply to the architect's land, but the lawyer (or any other lot owner) can enforce it as a reciprocal negative servitude. (A) is incorrect. While it is true that the architect's deed had no restrictions, those restrictions are binding if they are in her chain of title so as to give her notice of them. The restriction was in the deed from the developer to the investor, so the fact that it was omitted in the deed from the investor to the architect is of no significance. (B) is incorrect. While a covenant is normally only enforceable by the party receiving the promise (here, the developer), this is a situation of mutual rights of enforcement within a geographically defined area, a special situation that gives every lot owner in the area the right of enforcement, even though they did not directly receive the benefit of the promise. (D) is incorrect. The fact that gives the lawyer the right of enforcement is not just the fact that his deed contains the covenant, but that the same covenant was in all of the deeds from the developer, including the one to the architect's predecessor in title.

A husband and wife were both professors at the United States Naval Academy. The husband made a speech criticizing United States foreign policy with respect to a Middle Eastern country and was dismissed from his teaching position soon thereafter. Six months later, he accepted new employment in another state. The man's wife has commenced suit in federal court claiming that the Naval Academy violated her husband's right to due process and his right of free speech when it fired him. What is the most likely ground on which the court will dismiss the suit? (A) The political question doctrine, because foreign policy is in the domain of the executive branch. (B) The case is moot, because the husband has a new job. (C) The plaintiff has no standing. (D) There is no federal question presented.

(C) The most likely ground for dismissal is lack of standing. Except for very limited exceptions, the traditional rule is that a person has standing only to raise constitutional issues which affect MPQ 100 preview diagnostic exam and divider P.indd 63 12/22/2015 4:13:32 PM 64. MBE PREVIEW DIAGNOSTIC EXAM her personally. She cannot claim that a third person's constitutional rights were violated. The husband is the injured party and must assert his own rights. (A) is incorrect. The issue in this question is not the substance of America's foreign policy, which would be a political question, but the husband's First Amendment right to speak out about it, which is not a political question. Therefore, the court is not barred from litigating this issue under the political question doctrine. (B) is incorrect. There are damages to be assessed if the husband was improperly dismissed. Under Mt. Healthy Board of Education v. Doyle (1977), a public employee cannot be fired without a hearing for exercising the right to free speech, even if there is no property right in the job. The case is not moot because the husband has lost his job permanently. The case would be moot only if the husband had been reinstated to his former job at his former pay. (D) is incorrect. The subject matter of the suit—the dismissal of an employee for exercise of his First Amendment rights— presents a federal question.

A man put his leaf mulcher back into his garage after finishing his yard and then got into his car and drove away, accidentally leaving his garage door standing wide open. An envious neighbor raking his leaves across the street noticed the open garage and decided to help himself to the use of the man's leaf mulcher. The neighbor thought he could quickly finish his own leaves before the man returned without the man ever realizing that it had been used. However, the man pulled up in the driveway before the neighbor had taken three steps with the mulcher in hand. The neighbor quickly put down the mulcher and exited the garage, but the man was furious and called the police. Why will the neighbor be found not guilty of larceny? (A) The neighbor had not used the mulcher. (B) The garage door was left standing wide open when the neighbor took the mulcher. (C) The neighbor planned to return the mulcher before the man found out. (D) The neighbor was still in the garage when the man stopped him.

(C) The neighbor should be found not guilty of larceny because he did not have the requisite intent. Larceny consists of a taking and carrying away of tangible personal property of another by trespass, with intent to permanently deprive the person of his interest in the property. If the MPQ 100 preview diagnostic exam and divider P.indd 89 12/22/2015 4:13:33 PM 90. MBE PREVIEW DIAGNOSTIC EXAM defendant intends to return the property within a reasonable time and at the time of the taking has a substantial ability to do so, the unauthorized borrowing does not constitute larceny. Hence, the neighbor is not guilty of larceny because he planned to return the mulcher. (A) is incorrect because the fact that the neighbor had not yet used the mulcher would be irrelevant if he had formed the intent to steal it. (B) is incorrect because the fact that the garage door was open is irrelevant. The act of breaking into a dwelling is an element of burglary, not larceny. (D) is incorrect because the neighbor had taken possession of the mulcher and started to carry it away, which satisfies the asportation requirement. It is irrelevant that he had not yet exited the garage

A landlord entered into a written four-year lease with a tenant for an apartment in the landlord's apartment house. The tenant's lease, and all leases in the apartment house, prohibited the playing of musical instruments between 10 p.m. and 8 a.m. The lease required the tenant to pay the rent on a monthly basis. Two years into the lease, the tenant assigned the lease to a nurse with the landlord's permission. The nurse then assigned the lease to his brother with the landlord's permission. The brother went into possession. A neighboring tenant in the same apartment house insisted upon playing a trumpet in a loud manner between 2 a.m. and 4 a.m. The brother complained to the landlord without success. Unable to sleep each night, the brother abandoned his apartment after occupying it for two months. If the landlord sues the nurse for the rent due during the period after the nurse's brother left, what would be the nurse's best defense? (A) A breach of the landlord's covenant of quiet enjoyment. (B) Estoppel, because the landlord consented to a further assignment. (C) Lack of privity of estate. (D) Constructive eviction.

(C) The nurse is only an assignee of the tenant. The nurse is not directly contractually obligated to the landlord. Once an assignee of the nurse goes into possession of the premises (especially with the consent of the landlord), the privity of estate between the landlord and the nurse, the prior assignee, is terminated. An assignee has no contractual obligation under the original lease and is only liable for rent during the period of his estate, i.e., his possession. Thus, in this case, there is no longer any privity of estate between the nurse and the landlord, and the landlord cannot sue the nurse on the rent covenant. (A) is incorrect. The covenant of quiet enjoyment was not breached here because no one with superior title interfered with possession of the apartment. Moreover, the duty to provide quiet enjoyment is owed by the landlord only to the tenant and his successors in interest (i.e., those in privity of estate). Here, there is no privity of estate between the landlord and the nurse because the nurse is an assignee who is no longer in possession at the time of his brother's abandonment, and that is the nurse's best defense. (B) is incorrect. The defense of estoppel may be invoked where a party, acting in good faith, has been induced by the conduct of the adverse party to do something that it otherwise would not have done, that resulted in his harm, and that the adverse party had cause to know would so result in harm. These facts do not support the use of the theory of estoppel to avoid the application of landlord-tenant law, because the landlord has made no representations which were relied upon by the nurse to his detriment. (D) is incorrect, if only because there is a better defense available to the nurse. The defense of constructive eviction might be available to the nurse's brother, who was in privity of estate with the landlord during the month in question. However, it is less clear that it would be derivatively available to the nurse. Moreover, it is a difficult factual defense. The landlord must be shown to have the ability to control the noise and to have failed to exercise that control. The noise must be of sufficient magnitude to deprive the nurse's brother of the reasonable beneficial use of the apartment. The defense of lack of privity of estate is clear-cut and much more beneficial to the nurse.

A debt that a brother owed to his sister had an outstanding balance of $4,000 when the statute of limitations ran out. Urgently needing money but aware that she had no legal recourse, the sister asked her brother if he would be able to repay any part of the balance of the loan. The brother agreed in writing to assign to his sister a debt of $2,000 that was owed to him by his stepson and was coming due in a week, and the sister accepted. The brother informed his stepson of the assignment and instructed him to pay the sister. When the $2,000 debt became due, the stepson refused to pay her. If the sister brings an action against the stepson to collect the debt, will she likely prevail? (A) No, because partial assignments are ineffective. (B) No, because she cannot sue the stepson on the same debt that she is barred by the statute of limitations from recovering from her brother. (C) Yes, because the stepson may not raise the brother's statute of limitations defense on the original contract. (D) Yes, because the assignment is valid as a novation.

(C) The sister will likely prevail. While an obligor (the stepson here) may raise defenses on the obligation he owes to the assignor (the brother), he may not raise defenses that the assignor might have had against the assignee (the sister-creditor) on a different obligation as a means of avoiding his own obligation. Here, there are no apparent defenses to the stepson's liability to the brother for the $2,000 debt, and the stepson is not entitled to assert the brother's statute of limitations defense against the sister because the debt that the brother owed to his sister was a different obligation. MPQ 100 preview diagnostic exam and divider P.indd 83 12/22/2015 4:13:33 PM 84. MBE PREVIEW DIAGNOSTIC EXAM (B) is incorrect for the reason discussed above—she is suing the stepson on a different obligation. (A) is incorrect because the assignment of the stepson's debt to the brother is of the stepson's entire debt to him, even though it will only partially repay the money that the brother had owed to his sister. In any event, partial assignments are not invalid. Assignors may transfer some rights under the contract and retain others. (D) is incorrect because the assignment was not a novation. A novation substitutes a new party for an original party to the contract and requires the assent of all parties. If valid, it completely releases the original party. Here, the stepson did not assent to anything and no one was released, so there was no novation.

A successful stockbroker and his best friend, a land developer, were pleased that their only children were about to marry each other. The stockbroker and the developer wanted to get the marriage off to a good start. Thus, on July 1, the two agreed in writing that, on July 15, the stockbroker would deliver to the land developer a check for one-half of the market value of a two-bedroom condominium owned by the developer, and the developer would simultaneously deed the condominium to the bride and groom. On July 10, the newlyweds learned of the agreement but neither assented to it nor changed their position in reliance on it. On July 12, the stockbroker and the developer mutually agreed to cancel their agreement. If the newlyweds were to sue the developer and the stockbroker for breach of contract, would they prevail? (A) Yes, because their rights as intended beneficiaries vested when the contract was made on July 1 and a later revision was ineffective in affecting their rights. (B) Yes, because they received notice of the contract on July 10, and the rescission thereafter was ineffective. (C) No, because the stockbroker and developer had the right to mutually rescind their agreement. (D) No, because the newlyweds were incidental beneficiaries and had no right to enforce the contract.

(C) The stockbroker and developer would prevail. The rights of the newlyweds, as intended beneficiaries, would have vested if they had learned of the contract made for their benefit and either accepted it at the request of the contracting parties or changed their position in reliance on it. In this case, they did neither before the donors rescinded the contract, and therefore they have no right to enforce it. (A) is incorrect. This choice correctly states that the newlyweds are intended third-party beneficiaries of the contract made by their fathers. However, their rights do not vest as of the formation of the contract. Rather, they must at least assent to the contract at the request of the contracting parties or must change their position in reliance on the contract. Thus, rescission of the contract by the stockbroker and the developer was sufficient to terminate the rights of the newlyweds. (B) is incorrect. However, their rights do not vest merely because they have received notice of the contract. They must either accept the contract at the request of the contracting parties or must substantially rely on the contract to their detriment before they would be able to prevent the original contracting parties from rescinding the contract. (D) is incorrect. Because the parties to the contract contemplated that the newlyweds would receive a benefit under the terms of the agreement, the newlyweds were intended, and not incidental, beneficiaries.

A witness who was unable to speak was an eyewitness to a robbery in the park. He went to the police station shortly after the crime was committed, and when asked who committed the robbery, pointed to the defendant in a police lineup in which there were several men who were dressed like the defendant and were the defendant's size. The defendant was then indicted for robbery. The witness died before trial, and the sergeant in charge of the lineup desires to testify at the defendant's trial that the witness pointed to the defendant at the lineup. Is his testimony admissible? (A) Yes, as evidence of prior identification. (B) Yes, to prove the defendant committed the crime. (C) No, because the sergeant's testimony is hearsay. (D) No, because the defendant's counsel was not present at the lineup.

(C) The testimony is inadmissible. For purposes of the hearsay rule, a statement is defined by Federal Rule 801(a) as an oral or written assertion, or nonverbal conduct which is intended as an assertion. In this case, the witness's pointing at the defendant is a statement because it was intended by the witness to assert that the defendant was the criminal. The sergeant's testimony concerning the witness's identification is testimony concerning an out-of-court statement and is therefore hearsay. Because the statement comes within no exception to the hearsay rule, it is inadmissible. The evidence is not evidence of a prior identification because the witness is not present on the witness stand. (A) is incorrect. Evidence of a prior identification is nonhearsay under Federal Rule 801(d)(1)(c) in only a very narrow range of cases. For a prior identification to be nonhearsay, a witness on the stand must have made a prior statement identifying the defendant as someone he perceived earlier. The offered evidence does not come within that definition of nonhearsay because the witness is not present on the witness stand. The sergeant's testimony of the witness's assertive conduct is inadmissible hearsay. (B) is incorrect. The evidence is inadmissible because it is hearsay and comes within no hearsay exception. The evidence is not evidence of a prior identification because the witness is not present on the witness stand to testify to the prior identification. (D) is incorrect. The choice correctly states that the evidence is inadmissible. But it is inadmissible because it is hearsay, not because the defendant's counsel was not present at the lineup. Under constitutional principles, the defendant's counsel would not need to be present at a preindictment lineup such as this in order to have the prior identification be admitted at trial.

A defendant is on trial for robbing a liquor store. The store clerk testified that the defendant came into the store at about 11 p.m., pointed a black gun with a silver grip at him, and demanded that he give him all the money in the cash register. The clerk testified that the store was well lit and that the defendant was not wearing a mask. The defendant's attorney called the clerk's employer to testify that when the clerk gave her a report of the robbery, he told her that the defendant pointed a silver gun with a black grip at him. How should the trial judge rule on the admissibility of this testimony? (A) Admissible, because it tends to show that the clerk is an unreliable witness. (B) Admissible, because it has bearing on the clerk's truthfulness and veracity. (C) Inadmissible, because it is extrinsic evidence of a prior inconsistent statement on a collateral matter. (D) Inadmissible, because it is hearsay not subject to any exceptions

(C) The testimony should be found inadmissible. Extrinsic evidence of a prior inconsistent statement may not be used to impeach a witness upon a collateral matter. The clerk testified that he could recognize the defendant's face, and so the color of the gun is not material to any issue in the case under the facts given. (A) and (B) are incorrect because, even if the evidence does have some bearing on the clerk's credibility as a witness, it will be excluded because it will possibly confuse the issues or because it is a waste of time. (D) is not correct because the defendant is not seeking evidence to prove the truth of any material issue, but to impeach the clerk. The hearsay rule does not apply.

A man and woman who shared an apartment were arrested and charged with murder. The woman's attorney moved to have the defendants tried separately. The motion was granted. The woman was tried first and was acquitted of the crime. The man's trial began one month later. The man's attorney called a witness, who was now living with the man, to testify that, shortly after she was acquitted of murder, the woman had told the witness that she (the woman) had killed the victim because the victim owed her money, and that the man was not involved at all in the victim's death. Is the witness's testimony admissible? (A) Yes, as a statement by a co-conspirator. (B) Yes, as a declaration against penal interest. (C) No, because it is hearsay not within any exception. (D) No, because the woman is estopped from contradicting the jury verdict of acquittal.

(C) The witness's testimony would clearly qualify as hearsay. Because there is no applicable exception to the hearsay rule, his testimony is inadmissible. (A) is incorrect because the statement was not made during and in furtherance of the conspiracy, a requirement for admitting statements by a co-conspirator. (B) is incorrect because the woman has already been acquitted of the crime in question; hence, she is not subject to further prosecution. (D) is an incorrect statement of law.

A Hispanic woman was a citizen of a foreign country but had lived in the United States for 10 years. She had properly acquired resident alien status and regularly complied with all regulations pertaining to resident aliens, such as annual registration at the local post office. Several of her friends had been called for jury service and she thought it would be interesting to serve on a jury. However, after extensive research she discovered that no Hispanic female alien had ever served on a jury in the county where she lived; in fact, no person of that class had ever been in a venire panel. May she compel the county officials to include her and other persons of her class in venire panels through a lawsuit in federal district court? (A) Yes, because there is a consistent pattern of discrimination against persons of her class. (B) Yes, because alienage is a suspect category. (C) No, because certain privileges, such as jury service, may be constitutionally denied to aliens. (D) No, because she lacks standing, in that she is not involved in a case or controversy where the lack of jurors in her class might prejudice her right to a fair trial.

(C) The woman would not prevail in this lawsuit. The Supreme Court consistently has held that the government may restrict aspects of citizenship such as voting and, here, jury service to citizens. In fact, the Court has held that laws discriminating against aliens that are related to attributes of citizenship need only meet a rational basis test. (A) is incorrect because the discrimination is constitutional as long as it is rationally related to a legitimate government purpose. Restricting jury service to citizens would be regarded as such a purpose. (B) is incorrect because, although alienage is generally a suspect classification, only a rational basis test is used for discrimination related to aspects of citizenship. (D) is incorrect because she could claim an injury based on the fact that she was discriminated against. Those who claim a violation of equal protection of the law in that they have been discriminated against have standing to sue.

A pilot had given her 15-year-old son informal piloting lessons, including practice on takeoffs and landings on their farm's landing strip. Confident in her son's ability, she agreed to the son's request to fly by himself to his uncle's ranch, which also had a landing strip, to attend his cousin's high school graduation party. The son took off safely, but about five minutes later the aircraft's engine stopped running; the plane stalled and then crashed, killing the son. It was later determined that the engine ran out of fuel because a burst fuel line had been overlooked during a routine maintenance check of the pilot's plane the week before the crash. An airplane mechanic at the small private airport where the pilot went for fuel and maintenance had performed the maintenance check. The pilot brought a wrongful death action for the loss of her son against the mechanic's employer, the owner of the private airport. At trial, the owner of the airport raises a contributory negligence defense. Whose negligence, if found by the jury to be a contributing factor in causing the accident, will reduce the pilot's recovery? (A) The pilot in permitting her son to fly the airplane. (B) The son in his handling of the plane after the engine quit. (C) Both the pilot in permitting her son to fly and the son in handling the plane. (D) Either the pilot in permitting her son to fly or the son in handling the plane.

(D) Either the pilot's or the son's conduct may reduce the pilot's recovery in a jurisdiction applying pure comparative negligence. Recovery in a wrongful death claim is allowed only to the extent that the deceased could have recovered in the action had he lived. Because the pilot's wrongful death claim derives from her son's death, either the son's or the pilot's contributory negligence will diminish the claim. Thus, although (A) and (B) are correct statements, they are not as good as (D), which is more complete. (D) states that the negligence of either the pilot or the son will reduce recovery, and this is the correct rule. (C) is wrong because only one person need be negligent to reduce recovery.

A manufacturer of washing machines telephoned an engine company and asked for a price quote on an order for 250 washing machine motors built according to the specifications that the manufacturer had sent to the engine company the previous week. The company responded that it would sell the motors to the manufacturer at a cost of $20 apiece, with delivery in 30 days. The manufacturer agreed to the terms and instructed that its order be entered. The engine company immediately started work on the motors, and had made a substantial beginning on their manufacture (having completed 110 motors) when the manufacturer notified the company that it would not honor the contract. The engine company stopped work on the motors and sued the manufacturer, which raised the Statute of Frauds as a defense. The engine company responded that the specially manufactured goods exception takes the contract out of the Statute of Frauds. If the engine company loses, what is the most likely reason? (A) It had not substantially completed work on the contract. (B) It had completed work on less than half of the motors. (C) It stopped work on the motors before the job was completed. (D) It could have sold the motors in the ordinary course of its business.

(D) If the engine company loses, it will be because it could have sold the motors in the ordinary course of its business. This question concerns the specially manufactured goods exception to the U.C.C.'s Statute of Frauds. In order for the specially manufactured goods exception to apply, there MPQ 100 preview diagnostic exam and divider P.indd 62 12/22/2015 4:13:32 PM MBE PREVIEW DIAGNOSTIC EXAM 63. must be all three of the following elements: (i) the goods must be specially manufactured for the buyer, (ii) the seller must have started work on the goods or else entered into a commitment to purchase them from someone else, and (iii) the goods must not be sellable in the seller's ordinary course of business. If the goods are sellable, as (D) states, this exception does not apply. (A) and (B) are incorrect because the seller need only have started work on the goods, which the engine company had done. (C) is incorrect because the company was entitled to stop work after the manufacturer's repudiation.

A "wind farm" was constructed on the open land next to a gas station where a mechanic is employed. The huge windmills generated electrical power and oscillated at low speeds, resulting in very low-frequency but high-amplitude sound waves that caused extreme discomfort in a substantial minority of people. The mechanic is one of the people adversely affected by such sound waves; the vibrations from the windmills give him severe headaches and upset his immune system. If the mechanic brings an action for nuisance against the owners of the wind farm and loses, what is the most likely explanation? (A) Sound waves are not the kind of physical phenomenon on which liability for nuisance can be based. (B) The owners of the wind farm cannot generate power without producing the offending sound waves, despite their best efforts to find an alternative technology. (C) At least six out of 10 people are not affected by low-frequency sound waves. (D) The mechanic neither owns nor rents the gas station at which he works.

(D) If the mechanic loses, it will be because he does not own or rent the affected property. A private nuisance is a substantial and unreasonable interference with the use and enjoyment of real property. Because the mechanic does not own or rent the property where he works, he may not pursue a claim based on interference with that real property's use and enjoyment. (A public nuisance is an interference with the rights of the community at large—a situation not presented by the fact pattern.) (A) is wrong because sound waves may be a basis for a nuisance action. (C) is wrong because, if 40% of the people are bothered by the sound waves, they probably are a nuisance. It is not necessary for a majority of the population to be affected for there to be a nuisance, but more than one must be affected. The choice between (B) and (D) is a difficult one. Nuisance requires an unreasonable interference with the property, and if the facts in (B) were true, the defendants could arguably be acting reasonably. However, (D) is more clearly a basis for the mechanic to lose than (B).

The neighbors to the north and south of a poorly maintained house, a mechanic and a doctor, decided to "send a message" by placing a smoke bomb on the owner's front porch. The doctor and mechanic consulted with another neighbor, an army veteran who was known to be an explosives expert, to help them. The army veteran agreed, but unbeknownst to the two neighbors, the army veteran manufactured a bomb containing a lethal quantity of explosives because he wanted to make sure the owner had a good scare. He gave the bomb to the mechanic and doctor, along with a timed fuse. In the middle of the night, the mechanic set the bomb on the owner's porch. A few minutes later, a huge explosion rocked the neighborhood. Although nobody was injured, the blast blew away the front of the owner's house. Who, of the following, is likely to be found guilty of attempted murder? (A) The doctor, mechanic, and the army veteran. (B) The army veteran and the mechanic, but not the doctor. (C) The army veteran, but not the doctor or the mechanic. (D) Neither the mechanic nor the doctor nor the army veteran.

(D) None of these people would be guilty of attempted murder because, on these facts, they did not possess the specific intent to kill necessary for the crime of attempted murder. At common law, all attempts were specific intent crimes. Thus, before a criminal conviction for attempt would lie, it had to be established that the actor had the specific intent to engage in the behavior or to cause the harm prohibited by the criminal statute that the actor was charged with attempting to violate. Thus, before someone could be guilty of attempted murder, he had to possess the specific intent to kill. In this question, it is clear that the mechanic and the doctor did not have the specific intent to kill. Their intent was only to place a smoke bomb on the owner's porch. The situation with respect to the army veteran's intent is not quite as clear, but a fair reading of the facts indicates that he did not have a specific intent to kill. It is true that he made a bomb containing a lethal quantity of explosives, but the question goes on to state that he only intended to scare the slovenly owner. Had someone died in the blast, the army veteran could be found guilty of manslaughter due to his recklessness, or perhaps he could be found guilty of murder under the depraved heart theory. However, since no one died, he will not be guilty of attempted murder because there was no specific intent to kill. Thus (D) is correct, and (A), (B), and (C) are wrong.

The President of the United States issued an executive order banning conversation by all executive employees with members of the press unless prior permission had been obtained from a supervisor. Executive Department employees were subject to dismissal for violation of the order. An employee of the United States Department of Agriculture spoke to a reporter and told the reporter that the USDA sanctioned the approval of a dangerous pesticide because of payoffs made by lobbyists to a high-ranking USDA official. The newspaper printed the story and quoted the employee by name. After a hearing in line with civil service regulations, the employee was found to have violated the executive order barring unauthorized conversations with news media reporters, and the employee was fired. The employee brought suit in federal court for reinstatement, back pay, and other benefits and also prayed that the court should strike down the executive order as unconstitutional. If the employee's case reaches the United States Supreme Court, how should the Court rule? (A) The executive order is constitutional, because the President has plenary power to control Executive Department employees. (B) The executive order is constitutional, because government employment is a privilege and not a right. (C) The executive order is unconstitutional, because Congress, rather than the President, has authority to set the terms of federal employment. (D) The executive order is unconstitutional, because the President cannot broadly limit all executive employees' freedom of speech and association.

(D) The President's order is unconstitutional because it is too broad a limit on the freedom of speech and association of government employees. No government employees were allowed to have even a conversation with the press without the permission of a supervisor. Thus, it is unconstitutional. (A) is incorrect because the President's plenary power to control executive employees is limited by the Constitution. For example, the President cannot violate the First Amendment (or other constitutional guarantees) in regulating federal employees. (B) is incorrect because the Court no longer draws a distinction between rights and privileges. Furthermore, government employees would retain their First Amendment rights even if government employment were termed a "privilege." (C) is incorrect because the President has authority to enact some regulations for federal employees. The problem with this regulation is that it unduly restricts freedom of speech.

A State A citizen and a State B citizen were involved in a car accident. The State B citizen filed a negligence action against the State A citizen in federal district court, seeking $500,000 for injuries incurred in the accident. The State A citizen believes that she was not at fault and that the accident was caused by the negligence of the State B citizen. May the State A citizen assert in the pending action a negligence claim against the State B citizen, seeking $400,000 for the injuries the State A citizen suffered in the accident? (A) No, joinder is improper under these circumstances. (B) Yes, and the court has discretion to grant or deny the motion in the interest of justice. (C) Yes, as a counterclaim in the pending action, or she may assert it as an independent action. (D) Yes, but only as a counterclaim in the pending action and will be barred from asserting it as an independent action.

(D) The State A citizen must assert her claim. Because the State A citizen's claim arises from the same transaction or occurrence as the claim asserted against her in the pending action, the State A citizen's claim is a compulsory counterclaim which must be asserted in the pending action, or it is lost. As a result, (D) is correct and (A) is incorrect. (B) is incorrect in that it is not a discretionary call with the judge whether the claim may be filed as a counterclaim. (C) is incorrect because the claim may not be asserted as an independent action. FRCP 13 states that the pleader need not state a compulsory counterclaim if the subject matter of the compulsory counterclaim was already pending in another court. However, if the pleader has not filed suit on the subject matter of the compulsory counterclaim (which she has evidently not done here), she must file the compulsory counterclaim.

A landowner in fee simple signed a promissory note for $10,000 to a bank, and secured the note by a mortgage of her land to the bank. The mortgage was duly recorded. The landowner then sold the property to an attorney, who assumed and agreed to pay the mortgage to the bank on the land. The attorney did not make payments on the mortgage note to the bank. The bank, following appropriate statutory procedures, foreclosed the mortgage and gave notice to both the landowner and the attorney that it intended to sue for any deficiency. At the foreclosure sale, the property sold for $6,000. The bank now sues both the landowner and the attorney for $5,000, which is the remaining amount of the unpaid principal and interest on the note plus costs of foreclosure. Against which party will the bank be successful in obtaining a judgment? (A) Only the landowner. (B) Only the attorney. (C) Either the landowner or the attorney. (D) Both the landowner and the attorney

(D) The bank will be successful in obtaining a judgment against both the landowner and the attorney, although it may only collect once. When a grantee assumes the mortgage, the grantee expressly promises the grantor-mortgagor that he will pay the mortgage obligation as it becomes due. The mortgagee then becomes a third-party beneficiary of the grantee's promise to pay and can sue the grantee directly if the grantee fails to pay. After the assumption, the grantor-mortgagor becomes a surety who is secondarily liable to the mortgagee on the note if the grantee fails to pay. The landowner and the attorney are jointly liable, even though the attorney is primarily liable and the landowner is secondarily liable as a surety. Therefore, (A) and (B) are incorrect. (C) is incorrect. The bank is not required to choose between the landowner and the attorney and can obtain a judgment against both, although it may only collect once. Because the attorney assumed the mortgage obligation, the bank can sue the attorney, but it can also sue the landowner in the same action as a surety. The landowner and the attorney are jointly liable, even though the attorney is primarily liable and the landowner is secondarily liable as a surety.

An owner of land who was also a home contractor agreed in writing with a buyer to build a house to the buyer's specifications on his (the contractor's) land and then sell the house and lot to the buyer. The contract provided that the house was to be completed by March 1, with full payment due at that time. On March 1, the house was nearly complete, but due to delays in the delivery of materials, the contractor would need 20 more days to finish construction. On March 5, after discovering that the house had not yet been completed, the buyer notified the contractor in writing of her election to cancel the contract because of the contractor's failure to deliver the house by March 1. The contractor responded that, due to an unanticipated strike at his supplier's company, performance had been unforeseeably delayed and that the house would be ready by March 20. The buyer responded that she would no longer accept delivery of the house and land. The contractor then brought an action to recover damages for breach of contract. Who will prevail? (A) The buyer, because the express date listed in the contract indicates that time was of the essence. (B) The buyer, because delivery by March 1 was a condition precedent to the buyer's performance. (C) The contractor, because the strike was truly an unforeseeable intervening event. (D) The contractor, because the buyer received the substantial benefit of the bargain.

(D) The contractor will prevail because the buyer received the substantial benefit of her bargain. The failure to perform on time is a breach of contract, but in this case, it was a minor breach. Unless the nature of the contract is such as to make performance on the exact day agreed upon of vital importance, or the contract provides that time is of the essence, failure to perform at the stated time is not a material breach. Here, the home was nearly complete, and the delay was relatively short. The contract did not specify that time was of the essence; thus, the breach was minor. The remedy for a minor breach is damages; the aggrieved party is not relieved of her duty to perform. (A) is incorrect because merely stating a date for performance does not indicate that time is of the essence. There must be some explicit statement indicating that time is of the essence. (B) is incorrect. Although the delivery on March 1 is a condition precedent to the buyer's duty to pay, the condition is excused by substantial performance. The test for whether a party has substantially performed is the same as the one for assessing whether a breach is minor or material. Here the breach is minor, the contractor substantially performed, and the condition is excused. (C) is incorrect because an unforeseeable event does not discharge a party's duty to perform. A strike at the contractor's supplier does not rise to the level of impossibility or impracticability, which would discharge his duty to perform. The contractor could have procured the supplies elsewhere.

A bank executive was on trial for embezzling $10,000 from the bank where he worked. A key witness for the prosecution was called to testify, but on the stand he had difficulty remembering the specifics of a conversation he had with the executive regarding the executive's accounting procedures. To refresh the recollection of the witness, the prosecutor showed the witness a memorandum that the witness had written for his file, detailing the conversation. The witness reviewed the memorandum, and then testified that he recalled the conversation. He proceeded to testify about the specifics of the conversation. The defense counsel then asked that the memorandum be introduced into evidence. How should the court rule on the memorandum? (A) Exclude the evidence if it determines that the witness's memory was refreshed. (B) Exclude the evidence if it determines that a reasonable jury would conclude that the witness's memory was refreshed. (C) Admit the evidence if it determines that the witness's memory was not refreshed. (D) Admit the evidence if it was used to refresh the witness's recollection.

(D) The court should admit the evidence if it was used to refresh the witness's recollection. Because the witness read the notes, and then had an independent recollection of events, this qualifies as a present recollection refreshed. Normally a writing used to refresh is not placed into evidence. However, under Federal Rule of Evidence 612, if a writing is used to refresh the recollection of a witness, the opposing party has a right to introduce the document into evidence. Thus, (A) and (B) are incorrect. (C) is incorrect because, if the witness's memory was not refreshed, the memorandum would have to be introduced as a past recollection recorded. The proper foundation has not been laid for admission of a past recollection recorded.

While driving a new car he recently purchased from an authorized dealer, a consumer's car caught fire for an unknown reason. The fire not only damaged the car, but also injured the consumer. The consumer filed a products liability action against the manufacturer of the car in federal district court, seeking to recover compensatory damages for his injuries. The complaint alleged that parts of the electrical system in the car were defective and that the defects caused the fire. The manufacturer filed an answer that denied the existence of any defects and denied that any defects caused the fire, but stated that it lacked sufficient knowledge and information to know what caused the fire. During discovery, the consumer served an interrogatory on the manufacturer that asked the manufacturer to "identify and summarize all evidence of which the manufacturer [was] aware that indicated that the fire was not caused by a defect in the car." The manufacturer's response stated that it did not have, and was not aware of, any evidence indicating that the fire was not caused by a defect. Based on that interrogatory response, the consumer filed a motion for partial summary judgment on the issue of causation to establish that any fire was caused by defects. How should the court rule on the motion? (A) Grant the consumer's motion for summary judgment. (B) Grant the consumer's motion for summary judgment unless the manufacturer files affidavits or other evidence indicating that the fire was not caused by a defect. (C) The court should deny the consumer's motion, because it addresses an ultimate issue in the case. (D) The court should deny the consumer's motion, because, unless the consumer has presented evidence that a defect caused the fire, the manufacturer does not need to present evidence regarding causation.

(D) The court should deny the consumer's motion. The plaintiff must prove the elements of the prima facie case for her claim. Absent proof of an element of the prima facie case, summary judgment for the plaintiff is not appropriate. Thus, (A) is incorrect and (D) is correct. Furthermore, although a party is generally required to respond to a motion for summary judgment with affidavits, the facts here indicate that the consumer has not come forward with any evidence pertaining to the manufacturer's fault for the damage and injury. Thus, because the consumer has not properly supported his motion for summary judgment with relevant, admissible information, there is no need for manufacturer to produce any evidence to survive a motion for summary judgment. This makes (B) incorrect. (C) is an incorrect statement of the law. A motion for summary judgment may be granted even though the motion addresses an ultimate issue in the case.

A plaintiff sued a defendant in federal district court for breach of contract. Based on a preliminary investigation, the defendant's attorney believes that no legally enforceable contract ever existed. Unfortunately for the defendant, however, his attorney is unsure whether she can prove that point at trial. If a valid contract did exist, the defendant's attorney believes that the defendant did not breach it. She also thinks that she has a better chance of prevailing on that point. Which of the following is true regarding the defendant's ability to assert as defenses both that no contract existed and that the defendant did not breach the contract if one did exist? (A) The defendant may plead only one of these defenses because they are inconsistent. (B) The defendant may plead both defenses, but the plaintiff will then be able to have one defense stricken. (C) The defendant may plead both defenses, as long as he labels them as affirmative defenses. (D) The defendant may plead both defenses regardless of how they are labeled.

(D) The defendant may plead both defenses regardless of how they are labeled. The Federal Rules of Civil Procedure expressly permit inconsistent pleadings in the alternative or hypothetically, reasoning that the discovery process will operate to sort out the viable pleadings. For this reason, (A) and (B) are incorrect. (C) is incorrect because the Rules do not limit inconsistent claims or defenses to "affirmative" defenses.

After having much too much to drink at the corner bar, the defendant decided it was time to go home. However, because of her intoxication, she did not notice that she grabbed an expensive and distinctive fur coat instead of her inexpensive cloth coat from the bar's coat room. The owner of the fur coat, who noticed the defendant staggering out of the bar and down the street, ran out of the bar and shouted for the defendant to stop. A police officer on foot patrol heard the shout and immediately apprehended the defendant. If charged with larceny of the fur coat, should the defendant be found guilty? (A) Yes, because her mistake was unreasonable. (B) Yes, because voluntary intoxication is not a defense to a charge of larceny. (C) No, because the defendant did not reach a place of temporary safety and thus did not complete the crime. (D) No, because she was too intoxicated to realize the property did not belong to her.

(D) The defendant should be found not guilty. Larceny is a specific intent crime that requires that the defendant intend to permanently deprive the owner of his property. If the defendant can show that she really believed the property was her own, there is no intent to deprive the owner of his property, and therefore the defendant would not be guilty of larceny. This is true whether or not the mistake was reasonable or whether the mistake was attributable to intoxication. In the instant case, the defendant did not realize that she had taken another's coat, and thus did not have the specific intent to commit larceny. (A) is incorrect. A mistake of fact that negates the state of mind requirement for specific intent crimes need not be reasonable. Any mistake of fact, reasonable or unreasonable, that negates specific intent would be a defense. For much the same reason, (B) is incorrect. For specific intent crimes, intoxication may be a defense when it prevents the defendant from formulating the requisite state of mind. (C) is incorrect because it implies that the asportation requirement for larceny must be substantial. It need not be; any movement of the property is sufficient to satisfy the requirement. Here, the defendant walked out of the bar with the fur coat, which is a sufficient asportation. Exam Note: Reaching a place of temporary safety may be important in a question involving killings committed during the course of a felony because it may cut off liability; it has nothing to do with the asportation requirement for larceny

A bathroom fixture company, in a signed writing, contracted with the developer of an apartment building for the sale to the building of 50 identical sets of specified bathroom fixtures, 25 sets to be delivered on March 1, and the remaining 25 sets on April 1. The agreement did not specify the place of delivery, nor the time or place of payment. On March 1, the fixture company tendered 24 sets to the developer and explained that one of the 25 sets was damaged in transit from the manufacturer to the company. The fixture company also promised to deliver a replacement within five days. Which of the following statements is correct? (A) The developer is entitled to accept any number of the 24 sets, reject the rest, and cancel the contract both as to any rejected sets and the lot due on April 1. (B) The developer is entitled to accept any number of the 24 sets and to reject the rest, but is not entitled to cancel the contract as to any rejected sets or the lot due on April 1. (C) The developer must accept the 24 sets but is entitled to cancel the rest of the contract. (D) The developer must accept the 24 sets and is not entitled to cancel the rest of the contract.

(D) The developer must accept the 24 sets and is not entitled to cancel the rest of the contract. The ground for rejection of an installment contract is U.C.C. section 2-612(2), which provides that the buyer may reject any installment that is nonconforming only if it substantially impairs the value of that installment and cannot be cured. If the nonconformity can be cured and the seller gives adequate assurances of its cure, the shipment must be accepted by the buyer, provided that the defect is not such as to constitute a breach of the whole contract. (A) is incorrect. Under U.C.C. section 2-601, if defective goods arrive, the buyer normally has every right to reject them as unsatisfactory. But this is not the case if the defective shipment was part of an installment contract, as is the case here. (B) is incorrect for the reasons stated above. (C) is incorrect. Under U.C.C. section 2-612(3), there is a breach of the entire contract only when the nonconformity or default with respect to a single installment substantially impairs the value of the whole contract. Here, this is clearly not s

A pedestrian filed an action against a driver in federal district court, alleging negligence. The attorney for the driver has interviewed an eyewitness whose testimony will clearly indicate that the driver was at fault. Must the driver disclose the existence and identity of the eyewitness to the pedestrian? (A) No, because the identity of the eyewitness is protected from discovery under the work product doctrine. (B) No, because the driver is not likely to use the eyewitness as part of her case. (C) Yes, because the driver must disclose all witnesses who have discoverable information, even without a specific request. (D) Yes, but only in response to an appropriate interrogatory.

(D) The driver must disclose the identity of the witness in response to an appropriate interrogatory. Federal Rule 26(a) requires, as an initial disclosure, a party to reveal the name and contact information of individuals who are likely to have discoverable information and who the disclosing party may use to support his claims or defenses (unless the use would be solely for impeachment). After initial disclosures are made, discovery proceeds, and the parties may continue with discovery of nonprivileged information that is relevant to any party's claim or defense, including the names and contact information of any person who knows of any discoverable matter. Here, the eyewitness would not need to be disclosed as an initial disclosure because the driver obviously will not use the eyewitness to support the driver's claim or defense. However, the identity of the eyewitness would need to be disclosed eventually, assuming the pedestrian submits a proper discovery request. This makes (D) correct and (B) incorrect. (A) is incorrect because the work product doctrine does not prevent the disclosure of the existence of the eyewitness. Any materials generated by the attorney would probably be protected under the work product doctrine (unless a showing of substantial need and undue hardship can be made); however, the eyewitness's name and contact information would not be protected. (C) is an overbroad description of the initial disclosure requirements and is, thus, incorrect.

A drug dealer was convicted in federal court of possession of 10 kilos of cocaine with intent to distribute. She was sentenced to a prison term. Subsequently, a federal grand jury indicted the dealer under a separate statute for conspiracy to distribute the same 10 kilos of cocaine. She moved to dismiss the indictment. How should the court rule on her motion? (A) Grant it, because the Double Jeopardy Clause protects her against a second prosecution for the same criminal conduct. (B) Grant it, because the Due Process Clause protects her against double punishment for the same criminal conduct. (C) Deny it, because the Double Jeopardy Clause does not apply when the second prosecution is for violation of a separate statute. (D) Deny it, because each prosecution requires proof of an element that the other does not.

(D) The drug dealer's motion should be denied. The defense of double jeopardy does not apply if the second crime requires an element which is not an element of the first crime and vice versa. In this case, the crime of conspiracy has an element which is not in the possession crime, namely the agreement to commit the crime with another person, and it is lacking an element, namely the actual commission of the crime. (A) is incorrect because it is possible to commit two separate crimes by the same criminal conduct, such as conspiracy and the substantive offense. (B) is incorrect because it is the Double Jeopardy Clause of the Bill of Rights that affords protection against a subsequent prosecution, not the Due Process Clause. (C) is incorrect because even if a lesser included offense to the first crime were contained in a separate statute, the defense of double jeopardy would still apply

On April 4, a homeowner hired an electrician to evaluate his home's wiring. The electrician found the wiring to be faulty, called it a fire hazard, and urged the homeowner to fix it soon. That day, the homeowner entered into a written contract with the electrician, whereby the electrician agreed to repair the wiring for $1,600. The contract provided that the repair would take place prior to April 30, on a day when the homeowner would be home. On April 26, the work had yet to be done so the homeowner called the electrician to pin down a date. The electrician suggested April 29, and the homeowner agreed. On the morning of April 29, the electrician arrived ready to do the repair work. However, he discovered that no one was home, and he could not get inside. Earlier that morning, the homeowner had been rushed to the hospital for an emergency appendectomy, and the homeowner did not return home from the hospital until May 1. On that day, the faulty wiring caused a fire that severely damaged the house. Will the electrician succeed in a breach of contract action? (A) Yes, because the homeowner made and breached an implied-in-fact promise to make the house available to the electrician before April 30. (B) Yes, because any recovery by the electrician will be subject to a setoff by the homeowner on account of his damage from the fire. (C) No, because the electrician did nothing about the agreement between April 4 and April 26. (D) No, because the homeowner's obligations under the contract were subject to an express condition precedent that failed because of supervening impossibility.

(D) The electrician will not succeed because there was an express condition precedent that the repairs had to be performed on a day when the homeowner would be home, and the homeowner was unable to honor that commitment due to an unforeseeable supervening event. (C) is not the best answer. Although the electrician did nothing between April 4 and April 26, he did confirm the repair date of April 29 with the homeowner. The electrician would have honored the contract but for the supervening impossibility of the homeowner being unable to stay home. (A) is a false statement. The promise to make the house available was an express condition, not an implied-in-fact contract. (B) is wrong because impossibility discharges duties on both sides.

A toy makers' union, angry about poor working conditions and low wages, staged a nationwide strike just weeks before the holiday season. Larger toy sellers immediately hired independent toy makers to fulfill the traditionally increased demand for toys during the holidays. Enraged, striking toy makers committed acts of violence against independent toy makers and attempted to destroy shipments of independently made toys as they were being loaded off trucks at toy sellers' receiving docks. In response to the increasing violence, Congress met in emergency session and enacted a measure directing the President to send military troops to the affected areas to preserve order and to ensure the continued flow of commerce. Is this enactment constitutional? (A) Yes, under Congress's power to raise and support the armed forces. (B) Yes, under Congress's power to regulate commerce. (C) No, because it infringes on the President's authority to faithfully execute the laws of the United States. (D) No, because it infringes on the President's authority as Commander in Chief of the armed forces.

(D) The enactment appears to be an unconstitutional infringement on the President's authority as Commander in Chief. The President's role as Commander in Chief of the armed forces includes MPQ 100 preview diagnostic exam and divider P.indd 85 12/22/2015 4:13:33 PM 86. MBE PREVIEW DIAGNOSTIC EXAM extensive power to deploy the military against any enemy, foreign or domestic. Congress lacks such power. Therefore, this enactment directly infringes on the President's authority as Commander in Chief to make such orders as he deems proper with respect to the armed forces, and thus violates the doctrine of separation of powers. (A) is incorrect. Congress does have the power to raise and support an army, but the enactment here does not result in the appropriation of money to support the armed forces. Rather, it seeks to control their activities, and Congress has no such power. (B) is incorrect. Congress does have the power to regulate commerce, and when that power is combined with the Necessary and Proper Clause, Congress would have the power to enact legislation protecting the toy shipments at issue here. Nevertheless, the Commerce Clause, even when combined with the Necessary and Proper Clause, does not give Congress the power to violate other aspects of the Constitution. Ordering the President to send military troops violates the separation of powers doctrine because the Constitution gives the President the power as Commander in Chief of the armed forces. (C) states the correct result but is based on an incorrect rationale—the duty to execute the laws of the United States is an obligation, not a grant of authority.

A small group of terrorists hijacked a foreign airliner. Ten prominent bankers from the United States were among the airline passengers. The terrorists demanded that three individuals imprisoned in a state prison be released. Although these individuals were properly convicted of violation of state laws, the terrorists contended that they were political prisoners and that their convictions were merely a subterfuge to prevent them from inciting the local populace. The President of the United States agreed to release the three prisoners in return for the freedom of the 10 bankers. He issued official pardons and ordered the state governor to release the prisoners. The governor refused to do so, contending that the President had overreached his constitutional powers and had violated the concept of federalism. What is the best argument for the governor's position? (A) The President had no power to make an agreement with the terrorists because his treaty power extended only to dealings with the legitimate governments of other nations. (B) The Constitution immunizes a governor acting pursuant to a state constitution from inconsistent orders by a federal official. (C) The President's duty to see that the laws are faithfully executed prevents him from ordering state officials to release prisoners. (D) The President's constitutional power to grant pardons extends only to prisoners who have violated federal laws.

(D) The governor's best argument is based on separation of powers principles. Under Article II, Section 2 of the Constitution, the President has the power to "grant reprieves and pardon offenses against the United States, except in cases of impeachments." Thus, the President's pardon power is limited to violations of federal law. The President has no authority to pardon those convicted of state crimes or held in state custody. Thus, the governor is correct. The President lacks the authority to grant these pardons. Although the President's ability to negotiate treaties only extends to dealing with other nations, (A) is wrong because the President possesses other powers that would allow him to negotiate such deals. For example, the President's powers as Commander in Chief and other inherent powers (such as the ability to negotiate executive agreements) would justify this action. (B) is wrong because, under the doctrine of federal supremacy, federal law will govern state law when there is an inconsistency between the federal and the state laws. Therefore, the governor could be required to do an act that is a violation of state law, but a requirement of federal law. (C) is wrong because the President has the power to grant pardons to those convicted of violations of federal laws. The President's exercise of this authority is part of his faithful execution of the laws of the United States.

While walking down the street, a pedestrian accidentally bumped into a street performer. The performer told the pedestrian to pay attention to where she was going. The pedestrian got mad and slapped the performer across the face. The performer, afraid that the pedestrian would continue the assault and escalate it to the point of murdering him, pulled out a gun and shot the pedestrian, killing her instantly. In what type of state is the performer most likely to be convicted of manslaughter? (A) A state that recognizes the heat of passion resulting from adequate provocation as a mitigating circumstance. (B) A state that applies the doctrine of diminished responsibility. (C) A state that recognizes the misdemeanormanslaughter rule. (D) A state that recognizes the imperfect selfdefense doctrine.

(D) The imperfect self-defense doctrine applies to these facts. In states that apply the doctrine, murder is reduced to voluntary manslaughter when the defendant unreasonably, but honestly, believed in the necessity of responding with deadly force. In the instant case, the performer unreasonably, but apparently honestly, believed that the pedestrian would kill him. Thus, the performer would be guilty of voluntary manslaughter in such a jurisdiction. (A) is wrong. In order to have murder reduced to voluntary manslaughter based on a "heat of passion" theory, it must be shown that: (i) the provocation must have been one that would arouse sudden and intense passion in the mind of an ordinary person such as to cause him to lose self-control; (ii) the defendant was in fact provoked; (iii) there must not have been sufficient time to cool off; and (iv) the defendant did not in fact cool off. Adequate provocation generally consists of being subjected to a serious battery or threat of deadly force, or of discovering one's spouse in bed with another person. In the instant case, not only is it unlikely that a slap on the face would be considered a serious battery such that would provoke one to murder, the facts do not indicate that the performer acted in a fit of rage. (B) is wrong because there is no evidence of any diminution of the defendant's ability to reason. (C) is wrong. Although some states would call a killing committed during a misdemeanor "manslaughter," in the instant case there are no facts to indicate that the performer was in the process of committing a misdemeanor when he killed the pedestrian.

Twenty-five years ago, a property owner placed a large sewer line (to service a singlefamily house he built on the property) across a neighbor's property without the neighbor's permission. Four years ago, the owner tore down the house in preparation for the construction of a larger house on the land, and made an agreement with municipal authorities to take an easement across the neighbor's property and install a new sewer line to service the house. After a long delay, the municipal authorities failed to perform their agreement, and the owner finally constructed the house last year using the existing sewer line. The state has a 20-year statute for acquiring property interests by adverse use. If the neighbor attempts to enjoin the use of the sewer line to service the new house, will she prevail? (A) Yes, because the agreement with the municipal authorities estopped the owner from using the existing line to service the new house. (B) Yes, because the owner's tearing down of the house and nonuse of the sewer line for several years constituted an abandonment of the easement. (C) No, because the original sewer line was a license which ripened into an implied easement. (D) No, because the use was within the scope of the prescriptive easement acquired.

(D) The neighbor will not prevail. The owner has made an adverse use of the neighbor's land since the sewer line was first constructed across it. The fact that the owner tore down the house on his property does not stop the prescriptive period from running, because the mere existence of the sewer line was an adverse use and there was no intent to abandon it. Therefore, the owner's use ripened into a prescriptive easement 20 years after that use first began. Moreover, the use of the line to service the new house is within the scope of the easement by prescription because the same pipe in the same location will be used for the new house. (A) is incorrect. By the time the owner constructed the new house, he had maintained the sewer line across the neighbor's property for the statutory period and therefore had acquired an easement by prescription to continue to use that line. Once the owner obtained an easement by prescription over the neighbor's property, that property right persists despite any actions by the municipal authorities with regard to the neighbor's property, or any actions by the owner with regard to the authorities. (B) is incorrect. An easement is not terminated merely because it is not used for a long period of time. It can only be extinguished when the owner clearly demonstrates by physical action an intent to permanently abandon it. Here, abandonment could have arisen if the owner had been able to connect to a new sewer line, but that did not happen. (C) is incorrect. The owner never had a license. A license is a permissive use of property. The owner placed the sewer line across the neighbor's property without her permission. Moreover, the use never ripened into an implied easement. Such an easement can only occur when there is a division of a large parcel of property into smaller parcels and there is a reasonable necessity for the owner of one of the parcels to continue to use rights over another part of the larger parcel. There is no such division of property here. Instead, the adverse use by the owner ripened into an easement by prescription.

An intoxicated airline passenger collecting his bags after passing through security grabbed a garment bag from its owner because he mistakenly thought it was his own. The owner of the garment bag tried to take it back from the passenger. During the tug of war over the garment bag, the passenger knocked the owner to the floor, took the garment bag, and ran to his gate. The passenger was later arrested and charged with robbery. Should the passenger be found guilty? (A) Yes, because his intoxication was voluntary. (B) Yes, because mistake is no defense to robbery. (C) No, because he made no threats and was intoxicated. (D) No, because his mistake negated the required specific intent.

(D) The passenger should be acquitted. Robbery is larceny from a person by violence or intimidation. Larceny requires a specific intent to take personal property from the possession of another with intent to permanently deprive the other of his possessory interest. Because the passenger thought that the property belonged to him, his mistake of fact is a valid defense to the crimes of larceny and robbery. This is because the mistake negates the existence of a mental state (specific intent to steal) essential to the charged crime. (C) is incorrect. Robbery is larceny by either violence or intimidation. Knocking the owner to the floor constitutes the necessary violence, whether or not the passenger MPQ 100 preview diagnostic exam and divider P.indd 72 12/22/2015 4:13:32 PM MBE PREVIEW DIAGNOSTIC EXAM 73. used intimidation. Even though voluntary intoxication may be a defense to a specific intent crime (in those circumstances where the intoxication negates the specific intent), the better answer is (D), because the passenger's mistake of fact is a defense to the specific intent crime charged since he did not have the specific intent to steal. (A) is incorrect. As discussed above, although voluntary intoxication can be a defense to a specific intent crime, (D) is the better answer because the question does not establish that the mistake of fact was a product of the intoxication. (B) is incorrect. The passenger's mistake of fact negates the specific intent to steal required to convict him of robbery.

A gangster was threatening to inflict a "disfiguring injury" on a shopkeeper if the shopkeeper didn't accept "gang protection." The shopkeeper refused to pay and told the gangster to get out of his store. The gangster pulled out a switchblade and he flipped it at the shopkeeper. The knife thunked into the wall a few centimeters from the shopkeeper's left ear. "Oops. I slipped," said the gangster. "This is your last chance. Without my protection, you're a dead man." The gangster turned and began to walk from the store. The shopkeeper took a revolver from under the cash register, aimed it at the gangster, and shot. However, the shopkeeper missed, killing a delivery person instead. At the shopkeeper's subsequent trial for the murder of the delivery person, which of the following is the strongest argument that the shopkeeper can only be convicted of manslaughter? (A) The shopkeeper did not intend to kill or seriously injure the delivery person. (B) The shopkeeper was acting in self-defense when he shot at the gangster. (C) The shopkeeper was acting under duress when he shot the delivery person. (D) The shopkeeper was reacting to a sufficient provocation to mitigate the homicide

(D) The strongest argument is that the shopkeeper was reacting to a sufficient provocation. This is a very commonly tested MBE issue. (A) is incorrect. It is clear from the facts that the shopkeeper manifestly intended to kill or seriously injure the gangster, and this intent is transferred to the delivery person, who was killed by mistake. (B) is incorrect because any imminent threat of death or serious injury to the shopkeeper had passed at the time he shot at the gangster. (C) is incorrect because there is nothing in the facts that indicates duress. Therefore, (D) presents the strongest argument—that the gangster's intentional near miss with the knife was sufficient provocation to mitigate the intended homicide from murder to voluntary manslaughter.

A tool producer was in the business of manufacturing and selling various hardware. The owner of a retail hardware store ordered 1,000 hammers from the tool producer, using the standard form that the tool producer's sales personnel used. Shortly after the hardware store placed their order, the tool producer's headquarters caught fire. All the purchase and sale records were destroyed in the fire. Afterwards, the hardware store owner asserted that the salesperson had agreed that the price for the hammers would be $5,000. The tool producer maintained that the price was $5,500. The tool producer then sued the hardware store. At the trial, the tool producer's director of the order department took the stand. After establishing that the original order had been destroyed by fire, the director sought to testify that the price of the order was $5,500. How should the court rule on the admissibility of such testimony? (A) Inadmissible, because it is self-serving and untrustworthy. (B) Inadmissible, because of the best evidence rule. (C) Inadmissible, because it is not the best degree of secondary evidence. (D) Admissible

(D) The testimony is admissible. The best evidence rule requires the production of the original document when attempting to prove the contents of the document. [Fed. R. Evid. 1002] The director of the tool producer's order department seeks to testify that the price of the hardware store's order was $5,500. It appears that this testimony is based solely on the director having seen the standard tool producer's form on which the hardware store placed their order. Thus, the testimony is about the contents of a document—the tool producer's form that the store owner completed to order hammers. At first glance, therefore, it would appear that (B) is the correct answer, that the director's testimony would be inadmissible because of the best evidence rule. However, that rule does not require production of the original if the original has been lost or destroyed in good faith. [Fed. R. Evid. 104(1)] It appears that the order form was destroyed in good faith in a fire at the tool producer's headquarters. Thus, (B) is incorrect. (C) is also incorrect. If the original of a document cannot be produced to prove its contents, any form of secondary evidence can be substituted for the original. There is no requirement, for example, that a copy of the original be produced. Testimony as to the contents of the document is equally permissible. From the standpoint of the best evidence rule, then, (D) is the correct answer. The proposed testimony of the director of the tool producer's order department is admissible. (A) is incorrect because it confuses the question of admissibility of evidence with that of the weight that a fact finder should give to the evidence. That the proposed testimony is self-serving may render it unpersuasive, but not inadmissible. Also, if the self-serving nature of testimony rendered it inadmissible, then virtually any time a party testified in support of his proposition, the testimony would have to be excluded. The law of evidence has for centuries rejected such a broad rule requiring disqualification of witnesses. In addition, (A) states that the proposed testimony of the director of the tool producer's order department is untrustworthy. There is no factual basis in the question for reaching that conclusion.

A developer purchased a downtown lot on which she intended to have an office building constructed. She entered into a written agreement with a construction company to build a 50-story office tower for $250 million. The agreement specified that monthly progress payments of $20 million were to commence one month after the foundation was in place, with all remaining amounts due upon approval of the completed structure by the developer's architect. Construction commenced on March 10, and by April 1 the foundation was completed. On the night of April 30, a terrorist group detonated 500 pounds of explosives in the partially completed structure, totally destroying the building plus all of the construction equipment involved in the project. The construction company was forced into bankruptcy when its casualty insurer classified the loss as having occurred in an "undeclared war," and refused to replace the lost equipment. The developer engaged another construction company to construct the 50-story tower as originally planned at a price of $260 million. The trustee in bankruptcy has now brought an action against the developer to recover all payments due the company for the work completed prior to the terrorist act. How much is the trustee likely to recover? (A) $20 million. (B) The difference between $20 million and the cost to the construction company of the work completed at the time of the terrorist attack. (C) The reasonable value of the labor and materials expended by the construction company at the time of the terrorist attack. (D) Nothing.

(D) The trustee will probably recover nothing. Destruction of a building being built under a construction contract does not render the contract impossible or impracticable to perform and therefore does not discharge the contractor from his duties. The original construction company could not continue to perform because the loss of its equipment rendered it bankrupt. Therefore, it was in breach and not entitled to any contractual remedy. Neither could it recover in restitution or quasicontract. Unlike a contract to repair or remodel, no restitutionary or quasi-contract remedy is available when the subject matter of a construction contract is destroyed.

A large oil producer wanted to expand into the oil refinery business. It contacted a small oil refinery company and offered to purchase the company's refining facilities as well as its output of petroleum products from its remaining production facilities. Lawyers for the oil producer prepared a written agreement specifying the sale price for the smaller company's refining business. The agreement also included provisions permitting the oil producer, at its option, to purchase 100% of all petroleum products removed from the earth by the smaller company for the following 15 years, at current market prices. The agreement was signed by both parties. One month after the sale of the refining facilities, the oil producer sent a fax to the smaller company stating that it would be exercising its option to purchase the smaller company's oil output for the next 15 years. Is the written agreement between the oil producer and the smaller company enforceable? (A) No, because the provision permitting the oil producer to purchase all of the smaller company's output was not supported by any consideration. (B) No, because the provision permitting the oil producer to purchase all of the smaller company's output for 15 years was indefinite and uncertain. (C) Yes, as to the sale of the smaller company's refining facilities, but unenforceable as to the output provision. (D) Yes, in its entirety

(D) The written agreement is enforceable in its entirety. The initial contract had two components: the sale of the smaller company's refining facilities to the oil producer, and the oil producer's option to purchase the smaller company's output. Normally, the option provision would be illusory, because the oil producer's promise to purchase, at its option, 100% of the smaller company's output is not a promise to do anything and therefore is not consideration. However, there is other consideration here—the oil producer's purchase of the smaller company's refining facilities. The contract must be considered as a whole, and here the oil producer promised to purchase the smaller company's refining facilities. Moreover, the specification of the quantity of 100% of output is sufficiently certain and can be enforced because the quantity of output can be estimated from prior output. The specification of the price of "current market prices" is also sufficient. Thus, the written agreement is enforceable in its entirety, and (A), (B), and (C) are incorrect.

A landowner conveyed his parcel of land "to my sister and her heirs so long as it is used for residential purposes, but if it is ever used for other than residential purposes, then to the local community center." Five years later, the landowner died, devising all of his real estate to his friend and leaving his daughter as his only heir. The following year, the landowner's sister and the landowner's daughter entered into a contract with a third party to sell the parcel to him in fee simple for $100,000. After examining title, the third party refused to perform under the contract because he believed the sister and the daughter could not deliver good title. The jurisdiction follows the common law Rule Against Perpetuities and has a statute providing that all future estates and interests are alienable, descendible, and devisable in the same manner as possessory estates and interests. If the sister and the daughter sue the third party for specific performance, how should the court rule on their request? (A) Grant it, because the sister owns the parcel in fee simple. (B) Grant it, because the sister and the daughter together own the parcel in fee simple. (C) Deny it, because the local community center has a valid interest in the parcel. (D) Deny it, because the friend has a valid interest in the parcel.

(D) Their request for specific performance will be denied. With a future interests question that may raise the Rule Against Perpetuities, you must first label the interests granted. The landowner attempted to give a fee simple determinable to his sister ("to my sister and her heirs so long as it is used for residential purposes") followed by an executory interest in the community center ("but if it is ever used for other than residential purposes, then to the local community center"). If the grant to the community center is valid, the landowner would have retained nothing. But the grant over to the community center is void under the Rule Against Perpetuities. It will not vest until the property is used for nonresidential purposes, an event that could occur outside lives in being plus 21 years. If the interest in the community center is void, that leaves only the sister's fee simple determinable. That means the landowner must have kept a possibility of reverter, which always accompanies a fee simple determinable that is not followed by an executory interest. The landowner's possibility of reverter was devised to the friend under the landowner's will. Thus, both the sister and the friend have interests in the property, and the failure of the friend to join in the contract means no specific performance of it. (A) is incorrect. The sister has a fee simple determinable. When the Rule Against Perpetuities voided the attempted executory interest in the community center, it struck all the language following the comma: "but if . . ."; that left "to my sister and her heirs so long as it is used for residential purposes." The fee simple determinable is not a fee simple absolute; it is accompanied by the possibility of reverter in the grantor, and that person (or his successor) must join in the conveyance for the grantee to get a fee simple absolute. (B) is incorrect. The daughter owns nothing; the landowner devised his real property interests to the friend, so his heir takes none of it. The sister has a fee simple determinable, not a fee simple absolute. (C) is incorrect. The Rule Against Perpetuities voided the attempted executory interest in the local community center; it has nothing


Ensembles d'études connexes

Chemical Process Safety Questions

View Set

Data Loss Prevention and Mobile Endpoint Protection Graded Assessment

View Set

BLW 302 (Exam 1-2-3-4-5) Study Guide

View Set

Additional Chp. 13 Questions for Test 1

View Set

Greathouse nutrition final- ch. 14

View Set